Решение матриц гауссом: Онлайн калькулятор. Решение систем линейных уравнений. Метод Гаусса.

Содержание

Математика онлайн

Решение математики онлайн

Math34.biz – это современный способ решения математики, в том числе для сравнения самостоятельных решений с машинными вычислениями.

Пользование сервисом удобно и понятно каждому человеку, попавшему на сайт впервые. Сразу выбираете нужный калькулятор, вводите необходимые данные по вашей задаче и нажимаете кнопку «Решение». За считанные секунды ответ готов.

Чтобы не возникало трудностей с вводом данных, мы подготовили специальную статью Как вводить данные? Помимо правил написания формул и чисел, в ней вы можете увидеть, как правильно вводятся различные константы и математические функции.

О калькуляторах

По мере возможности добавляются новые математические калькуляторы. На сегодняшний день их более 85.

Если не удалось найти нужный калькулятор, которым может быть решена ваша математическая задача, или есть предложение по улучшению имеющегося калькулятора, пожалуйста, сообщите об этом на почту [email protected]

Преимущества

1. Бесплатно
Решение математики онлайн не будет вам стоить ни копейки. Наш сервис абсолютно бесплатный и доступен любому пользователю интернета.

2. Без регистрации
Для пользования калькуляторами не требуется регистрации на сайте, отнимая время на заполнение почтовых ящиков и других личных данных.

3. Подробные решения
На многие задачи вы получите пошаговый развернутый ответ, что позволяет понять, каким образом было получено решение задачи.

4. Разные способы решения задач
Для популярных калькуляторов доступны разные методы решения задач, если они применимы, что позволяет, во-первых, лучше понять, как решается задача известным вам способом, а, во-вторых, научиться решать ту же самую задачу альтернативными методами.

5. Точность вычислений
В полученном ответе не приходится сомневаться, ведь мощная система расчета обеспечивает высокую точность при решении математических задач онлайн.

Однако, мы не исключаем возможность каких-либо ошибок, ведь известно, что алгоритмы пишутся хотя и очень умными, но всё же людьми. В случае обнаружения ошибки, пожалуйста, не поленитесь и сообщите нам о ней.

Системы линейных уравнений. Метод Гаусса

Рассмотрим систему линейных уравнений:

   

С этой системой связываются две матрицы: матрица коэффициентов

   

и расширенная матрица — с присоединенными свободными членами:

   

Элементарными преобразованиями системы линейных уравнений называются:

1. умножение уравнения на отличное от нуля число;

2. прибавление к одному уравнению любого другого, умноженного на любое число;

3. перестановка уравнений местами.

Теорема. Любая система линейных уравнений с помощью элементарных преобразований и, может быть, изменением нумерации неизвестных, может быть приведена к системе с трапециевидной матрицей.

Доказательство. Проводим элементарные преобразования только над строками матрицы , как в доказательстве теоремы о ранге матрицы. Возможно, при этом придется изменить нумерацию неизвестных. Приводим систему уравнений к виду

   

Если хотя бы одно из чисел отлично от нуля, то данная система уравнений решений не имеет (несовместна). Если же все они равны нулю, то последние равенств не несут никакой информации и могут быть отброшены. Тогда, если , то неизвестным можно придавать произвольные значения, а неизвестные находим из решения системы с треугольной матрицей

   

Эту систему удобно решать, определив из -го уравнения , затем из -го и т.д. Таким образом, можно выразить переменные через и получить общее решение системы. Если , то система (в случае совместности) имеет единственное решение.

Преобразование системы уравнений к системе с трапециевидной матрицей называется прямым ходом метода Гаусса. Последовательное вычисление неизвестных в порядке называется обратным ходом.

Пример. Решить систему линейных уравнений

   

Решение. Составим расширенную матрицу системы:

   

Первую строку умножим на 3 и вычтем из второй. Затем первую строку умножим на 2 и вычтем из третьей. Получим

   

Далее вторую строку прибавим к третьей и отбросим нулевую строку, получим

   

Запишем полученные уравнения:

   

Из второго уравнения выразим :

   

Полученное выражение подставляем в первое уравнение и выражаем из него :

   

Ответ. Общее решение данной системы:

   

Задачи.

1. Решите систему линейных уравнений

   

2. Решите систему линейных уравнений

   

3. Решите систему линейных уравнений

   

Решение СЛАУ и матрицы в Matlab

Доброго времени суток, читатели! Сегодня мы поговорим о матрицах в Matlab, об их применении в решении систем линейных алгебраических уравнений. Подробно разберем методы решения, и для этого необходимо знание нескольких базовых алгоритмов.

Также стоит отметить, что у каждого алгоритма, которым мы будем искать решение СЛАУ в Matlab, своя скорость нахождения этого решения, наличие или отсутствие условия выполнения алгоритма и т.д.

В традициях нашего сайта разберём на примере:

Решить систему линейных уравнений:

4*a + b - c = 6
a - b + c = 4
2*a - 3*b - 3*c = 4

Метод обратной матрицы в Matlab

Начнем с достаточно распространенного метода. Его суть состоит в том, что сначала необходимо выписать коэффициенты при a, b и c (то есть те коэффициенты, которые находятся слева) в одну матрицу, а свободный член (то есть то, что справа) в другую.

В итоге у нас получится 2 матрицы:

A=[4  1 -1; 1 -1  1; 2 -3 -3];   % коэффициенты
B=[6; 4; 4];

Для реализации этого метода (и следующих методов тоже) требуется одно условие: чтобы определитель матрицы, составленной из коэффициентов левой части не был равен нулю. Проверка на определитель:

det(A)

Вывод: 30

После проверки условия можем перейти к следующему шагу: нахождение обратной матрицы. В Matlab для этого используется оператор inv.
А само решение СЛАУ в Matlab находится как перемножение найденной обратной матрицы на матрицу свободных членов:

x=inv(A)*B

Вывод:
2
-1
1

Мы получили 3 значения, которые и соответствуют нашим коэффициентам: то есть a = 2, b = -1, c = 1. Можете проверить, подставив полученные ответы в исходную систему, и убедиться, что мы решили СЛАУ правильно.

Также следует отметить, что матрицы нужно перемножать именно, как сделали мы, то есть слева обратная матрица, справа матрица свободных членов.

Если вы не все поняли, то советую вам почитать нашу статью по основам Matlab.

Метод Гаусса

Метод Гаусса в Matlab реализуется достаточно просто: для этого нам нужно всего лишь изучить один новый оператор.
(\) - левое деление.
При следующей записи:

x = A\B

Вывод:
2
-1
1

Мы получим ответы на нашу исходную систему. Только заметьте, мы решили СЛАУ стандартным набором функций в Matlab, и желательно этот оператор использовать когда матрица коэффициентов квадратная, так как оператор приводит эту матрицу к треугольному виду. В других случаях могут возникнуть ошибки.

Метод разложения матрицы

Теперь поговорим о разложении матрицы. Нахождение решения через разложение матрицы очень эффективно. Эффективность обусловлена скоростью нахождения решения для данного вида систем и точностью полученных результатов.

Возможны следующие разложения:

  • разложение Холецкого
  • LU разложение
  • QR разложение

Разберём решение через LU и QR разложение, так как в задачах чаще всего встречается задание на решение именно через такие разложения.

Основное отличие этих двух разложений: LU разложение применимо только для квадратных матриц, QR — возможно и для прямоугольных.

LU разложение

Решим выше предложенную задачу через LU разложение:

[L, U] = lu(A);

Вывод:

L =
    1       0     0
    0.25    1     0
    0.5     2.8   1

U =
    4     1     -1
    0    -1.25   1.25
    0     0     -5

Затем:

y = L\B;
x = U\y

Вывод:

2
-1
1

QR разложение

И через QR разложение соответственно:

[Q, R] = qr(A);
x = R\(Q'*B)

Вывод:

2.0000
-1.0000
1.0000

Отметим, что апостроф (  '  ) после Q означает транспонирование.

Стандартные функции Matlab

Так же Matlab предлагает функцию linsolve, с помощью которой возможно решить систему линейных алгебраических уравнений. Выглядит это так:

x = linsolve(A,B)

Вывод:

2
-1
1

Как видите, ничего сложного тут нет, на то они и стандартные функции Matlab.

Повторение

Итак, сегодня мы с вами изучили несколько методов для решения СЛАУ в Matlab, как с помощью матриц, так и с помощью стандартных функций. Давайте их повторим на другом примере:

Решить систему линейных уравнений:
6*a - b - c = 0
a - 2*b + 3*d = 0
3*a - 4*b - 4*c = -1

A=[6 -1 -1; 1 -2 3; 3 -4 -4];
B=[0; 0; -1];
  • Методом обратной матрицы:
x=inv(A)*B

Вывод:
    0.0476
    0.1810
    0.1048
  • Методом Гаусса:
  • x = A\B
    
    Вывод:
        0.0476
        0.1810
        0.1048
    
  • LU разложение:
  • [L, U] = lu(A);
    y = L\B;
    x = U\y
    
    Вывод:
        0.0476
        0.1810
        0.1048
    
  • QR разложение:
  • [Q, R] = qr(A);
    x = R\(Q'*B)
    
    Вывод:
        0.0476
        0.1810
        0.1048
    

    На этом я с вами попрощаюсь, надеюсь, вы научились применять матрицы в Matlab для решения СЛАУ.

    Поделиться ссылкой:

    Похожее

    Страница не найдена — ПриМат

    © 2012-2016: Нохум-Даниэль Блиндер (11), Анастасия Лозинская (10), Юлия Стерлянко (8), Денис Стехун (8), Елизавета Савицкая (8), Игорь Любинский (8), Олег Шпинарев (7), Александр Базан (7), Валентин Малявко (7), Анна Чалапчий (7), Константин Берков (7), Татьяна Корнилова (6), Влад Радзивил (6), Максим Швандт (6), Людмила Рыбальченко (6), Кирилл Волков (6), Александр Земсков (5), Марина Чайковская (5), Екатерина Шибаева (5), Мария Корень (5), Анна Семененко (5), Мария Илларионова (5), Сергей Черкес (5), Алиса Ворохта (5), Валерия Заверюха (5), Елизавета Снежинская (5), Вадим Покровский (5), Даниил Радковский (5), Влад Недомовный (5), Александр Онищенко (5), Андрей Метасов (5), Денис Базанов (5), Александр Ковальский (5), Полина Сорокина (4), Кирилл Демиденко (4), Дмитрий Стеценко (4), Александр Рапчинский (4), Святослав Волков (4), Иван Мясоедов (4), Владислав Стасюк (4), Алёна Гирняк (4), Николай Царев (4), Валентин Цушко (4), Павел Жуков (4), Роман Бронфен-Бова (4), Артём Романча (4), Анна Шохина (4), Иван Киреев (4), Никита Савко (4), Кондрат Воронов (4), Алина Зозуля (4), Иван Чеповский (4), Артем Рогулин (4), Игорь Чернега (4), Даниил Кубаренко (4), Ольга Денисова (4), Татьяна Осипенко (4), Яков Юсипенко (4), Ольга Слободянюк (4), Руслан Авсенин (4), Екатерина Фесенко (4), Дмитрий Заславский (4), Алина Малыхина (4), Андрей Лисовой (4), Кирилл Сплошнов (3), Карина Миловская (3), Дмитрий Козачков (3), Мария Жаркая (3), Алёна Янишевская (3), Александра Рябова (3), Дмитрий Байков (3), Павел Загинайло (3), Томас Пасенченко (3), Виктория Крачилова (3), Таисия Ткачева (3), Владислав Бебик (3), Илья Бровко (3), Максим Носов (3), Филип Марченко (3), Катя Романцова (3), Илья Черноморец (3), Евгений Фищук (3), Анна Цивинская (3), Михаил Бутник (3), Станислав Чмиленко (3), Катя Писова (3), Дмитрий Дудник (3), Дарья Кваша (3), Игорь Стеблинский (3), Артем Чернобровкин (3), Виктор Булгаков (3), Дмитрий Мороз (3), Богдан Павлов (3), Игорь Вустянюк (3), Андрей Яроцкий (3), Лаура Казарян (3), Екатерина Мальчик (3), Анатолий Осецимский (3), Иван Дуков (3), Дмитрий Робакидзе (3), Вячеслав Зелинский (3), Данила Савчак (3), Дмитрий Воротов (3), Стефания Амамджян (3), Валерия Сиренко (3), Георгий Мартынюк (3), Виктор Иванов (3), Вячеслав Иванов (3), Валерия Ларикова (3), Евгений Радчин (3), Андрей Бойко (3), Милан Карагяур (3), Александр Димитриев (3), Иван Василевский (3), Руслан Масальский (3), Даниил Кулык (3), Стас Коциевский (3), Елизавета Севастьянова (3), Павел Бакалин (3), Антон Локтев (3), Андрей-Святозар Чернецкий (3), Николь Метри (3), Евелина Алексютенко (3), Константин Грешилов (3), Марина Кривошеева (3), Денис Куленюк (3), Константин Мысов (3), Мария Карьева (3), Константин Григорян (3), Колаев Демьян (3), Станислав Бондаренко (3), Ильдар Сабиров (3), Владимир Дроздин (3), Таня Спичак (2), Радомир Сиденко (2), Владислав Шиманский (2), Илья Балицкий (2), Алина Гончарова (2), Владислав Шеванов (2), Андрей Сидоренко (2), Александр Мога (2), Юлия Стоева (2), Александр Розин (2), Надежда Кибакова (2), Майк Евгеньев (2), Евгений Колодин (2), Денис Карташов (2), Александр Довгань (2), Нина Хоробрых (2), Роман Гайдей (2), Антон Джашимов (2), Никита Репнин (2), Инна Литвиненко (2), Яна Юрковская (2), Гасан Мурадов (2), Богдан Подгорный (2), Алексей Никифоров (2), Настя Филипчук (2), Гук Алина (2), Михаил Абабин (2), Дмитрий Калинин (2), Бриткариу Ирина (2), Никита Шпилевский (2), Алексей Белоченко (2), Юлиана Боурош (2), Никита Семерня (2), Владимир Захаренко (2), Дмитрий Лозинский (2), Яна Колчинская (2), Юрий Олейник (2), Кирилл Бондаренко (2), Елена Шихова (2), Татьяна Таран (2), Наталья Федина (2), Настя Кондратюк (2), Никита Гербали (2), Сергей Запорожченко (2), Николай Козиний (2), Георгий Луценко (2), Владислав Гринькив (2), Александр Дяченко (2), Анна Неделева (2), Никита Строгуш (2), Настя Панько (2), Кирилл Веремьев (2), Даниил Мозгунов (2), Андрей Зиновьев (2), Андрей Данилов (2), Даниил Крутоголов (2), Наталия Писаревская (2), Дэвид Ли (2), Александр Коломеец (2), Александра Филистович (2), Евгений Рудницкий (2), Олег Сторожев (2), Евгения Максимова (2), Алексей Пожиленков (2), Юрий Молоканов (2), Даниил Кадочников (2), Александр Колаев (2),

    Решение систем линейных уравнений методом Гаусса


    Теория

    Классическим методом решения систем линейных алгебраических уравнений является метод Гаусса (метод исключений Гаусса).
    Суть метода — это последовательное исключение неизвестных, т.е. когда с помощью элементарных
    преобразований система уравнений приводится к равносильной системе ступенчатого
    вида, из которой последовательно, начиная с последних переменных, находятся все остальные переменные.

    Матрица, составленная из все ai,j, называется основной матрицей системы. Если
    к этой матрице добавить вектор столбец, составленный из bi, то такая матрица называется расширенной
    матрицей системы.

    Теорема Кронекера-Капелли (условие совместности системы): системат совместна тогда и только тогда,
    ранг ее основной матрицы равен рангу ее расширенной матрицы.

    Алгоритм решения СЛАУ методом Гаусса подразделяется на два этапа:

    • На первом этапе (прямой ход) система приводится ступенчатой или треугольной форме. Вычтем из второго
      уравнения системы первое, умноженное на такое число, чтобы обнулился коэффициент при x1.
      Затем таким же образом вычтем первое уравнение из третьего, четвертого и т.д. Тогда исключаются все
      коэффициенты первого столбца, лежащие ниже главной диагонали. Затем при помощи второго уравнения
      исключим из третьего, четвертого и т.д. уравнений коэффициенты второго столбца. Последовательно
      продолжая этот процесс, исключим из матрицы все коэффициенты, лежащие ниже главной даигонали.
    • На втором этапе (обратный ход) выражаем все получившиеся базисные переменные через небазисные и
      построим фундаментальную систему решений. Если все переменные являются базисными, то получим
      единственное решение системы линейных уравнений. Эта процедура начинается с последнего уравнения,
      из которого выражают соответствующую базисную переменную (а она там всего одна) и подставляют
      в предыдущие уравнения, и так далее, поднимаясь по «ступенькам» наверх. Каждой строчке
      соответствует ровно одна базисная переменная, поэтому на каждом шаге, кроме последнего (самого верхнего),
      ситуация в точности повторяет случай последней строки.

    Метод Гаусса

    Пусть задана система линейных алгебраических уравнений, которую необходимо решить (найти такие значения неизвестных хi, что обращают каждое уравнение системы в равенство).

    Мы знаем, что система линейных алгебраических уравнений может:

    1) Не иметь решений (быть несовместной).
    2) Иметь бесконечно много решений.
    3) Иметь единственное решение.

    Как мы помним, правило Крамера и матричный метод непригодны в тех случаях, когда система имеет бесконечно много решений или несовместна. Метод Гауссанаиболее мощный и универсальный инструмент для нахождения решения любой системы линейных уравнений, который в каждом случае приведет нас к ответу! Сам алгоритм метода во всех трёх случаях работает одинаково. Если в методах Крамера и матричном необходимы знания определителей, то для применения метода Гаусса необходимо знание только арифметических действий, что делает его доступным даже для школьников начальных классов.

    Преобразования расширенной матрицы (это матрица системы —  матрица, составленная только из коэффициентов при неизвестных, плюс столбец свободных членов) системы линейных алгебраических  уравнений в методе Гаусса:

    1) строки матрицы можно переставлять местами.

    2) если в матрице появились (или есть) пропорциональные (как частный случай – одинаковые) строки, то следует удалить из матрицы все эти строки кроме одной.

    3) если в матрице в ходе преобразований появилась нулевая строка, то ее также следует удалить.

    4) строку матрицы можно умножить (разделить) на любое число, отличное от нуля.

    5) к строке матрицы можно прибавить другую строку, умноженную на число, отличное от нуля.

    В методе Гаусса элементарные преобразования не меняют решение системы уравнений.

    Метод Гаусса состоит из двух этапов:

    1. «Прямой ход» — с помощью элементарных преобразований привести расширенную матрицу системы линейных алгебраических уравнений к «треугольному» ступенчатому виду: элементы расширенной матрицы, расположенные ниже главной диагонали, равны нулю (ход «сверху-вниз»). Например, к такому виду:

    Для этого выполним следующие действия:

    1)   Пусть мы рассматриваем первое уравнение системы линейных алгебраических уравнений и коэффициент при х1 равен К. Второе, третье и т.д. уравнения преобразуем следующим образом: каждое уравнение (коэффициенты при неизвестных, включая свободные члены) делим на коэффициент при неизвестном х1, стоящий в каждом уравнении, и умножаем на К. После этого из второго уравнения (коэффициенты при неизвестных и свободные члены) вычитаем первое. Получаем при х1 во втором уравнении коэффициент 0. Из третьего преобразованного уравнения вычитаем первое уравнение, так до тех пор, пока все уравнения, кроме первого, при неизвестном х1 не будут иметь коэффициент 0.

    2)   Переходим к следующему уравнению. Пусть это будет второе уравнение и коэффициент при х2 равен М. Со всеми «нижестоящими» уравнениями поступаем так, как описано выше. Таким образом, «под» неизвестной х2 во всех уравнениях будут нули.

    3)   Переходим к следующему уравнению и так до тех пора, пока не останется одна последняя неизвестная и преобразованный свободный член.       

    1. «Обратный ход» метода Гаусса – получение решения системы линейных алгебраических уравнений (ход «снизу-вверх»). Из последнего «нижнего» уравнения получаем одно первое решение – неизвестную хn. Для этого решаем элементарное уравнение А*хn = В. В примере, приведенном выше, х3 = 4. Подставляем найденное значение в «верхнее» следующее уравнение и решаем его относительно следующей неизвестной. Например, х2 – 4 = 1, т.е. х2 = 5. И так до тех пор, пока не найдем все неизвестные.

    Пример.

    Решим систему линейных уравнений методом Гаусса, как советуют некоторые авторы:

    Запишем расширенную матрицу системы и с помощью элементарных преобразований приведем ее к ступенчатому виду:

    Смотрим на левую верхнюю «ступеньку». Там у нас должна быть единица. Проблема состоит в том, что в первом столбце единиц нет вообще, поэтому перестановкой строк ничего не решить. В таких случаях единицу нужно организовать с помощью элементарного преобразования. Обычно это можно сделать несколькими способами. Поступим так: 
    1 шаг. К первой строке прибавляем вторую строку, умноженную на –1. То есть, мысленно умножили вторую строку на –1 и выполнили сложение первой и второй строки, при этом вторая строка у нас не изменилась.

    Теперь слева вверху «минус один», что нас вполне устроит. Кто хочет получить +1, может выполнить дополнительное действие: умножить первую строку на –1 (сменить у неё знак).

    Дальше алгоритм работает уже по апробированной методике:

    2 шаг. Ко второй строке прибавили первую строку, умноженную на 5. К третьей строке прибавили первую строку, умноженную на 3.

    3 шаг. Первую строку умножили на –1, в принципе, это для красоты. У третьей строки также сменили знак и переставили её на второе место, таким образом, на второй «ступеньке у нас появилась нужная единица.

    4 шаг. К третьей строке прибавили вторую строку, умноженную на 2.

    5 шаг. Третью строку разделили на 3.

    Признаком, который свидетельствует об ошибке в вычислениях (реже – об опечатке), является «плохая» нижняя строка. То есть, если бы у нас внизу получилось что-нибудь вроде (0 0 11 |23), и, соответственно, 11x3 = 23, x3 = 23/11, то с большой долей вероятности можно утверждать, что допущена ошибка в ходе элементарных преобразований.

    Выполняем обратный ход, в оформлении примеров часто не переписывают саму систему, а уравнения «берут прямо из приведенной матрицы». Обратный ход, напоминаю, работает «снизу вверх». В данном примере получился подарок:

    x3 = 1
    x2 = 3
    x1 + x2 – x3 = 1, следовательно x1 + 3 – 1 = 1, x1 = –1

    Ответ: x1 = –1, x2 = 3, x3 = 1.

    Решим эту же систему по предложенному алгоритму. Получаем

    4   2   –1   1
    5   3   –2   2
    3   2   –3   0

    Разделим второе уравнение на 5, а третье – на 3. Получим:

    4   2       –1    1
    1   0.6    –0.4  0.4
    1   0.66  –1    0

    Умножим второе и третье уравнения на 4, получим:

    4    2       –1    1
    4    2,4    –1.6  1.6
    4    2.64  –4    0

    Вычтем из второго и третьего уравнений первое уравнение, имеем:

    4    2       –1      1
    0    0.4    –0.6   0.6
    0    0.64  –3    –1

    Разделим третье уравнение на 0,64:

    4    2      –1              1
    0    0.4   –0.6           0.6
    0    1      –4.6875    –1.5625

     Умножим третье уравнение на 0,4

    4    2       –1              1
    0    0.4    –0.6           0.6
    0    0.4    –1.875     –0.625

    Вычтем из третьего уравнения второе, получим «ступенчатую» расширенную матрицу: 

    4    2      –1              1
    0    0.4   –0.6           0.6
    0    0      –1.275      –1.225

    Таким образом, так как в процессе вычислений накапливалась погрешность, получаем х3 = 0,96 или приблизительно 1.

    х= 3 и х= –1.

    Решая таким образом, Вы никогда не запутаетесь в вычислениях и не смотря на погрешности вычислений, получите результат.

    Такой способ решения системы линейных алгебраических уравнений легко программируем и не учитывает специфические особенности коэффициентов при неизвестных, ведь на практике (в экономических и технических расчетах) приходиться иметь дело именно с нецелыми коэффициентами.

    Желаю успехов! До встречи на занятиях! Репетитор Дмитрий Айстраханов.

    © blog.tutoronline.ru,
    при полном или частичном копировании материала ссылка на первоисточник обязательна.

    МЕТОД ГАУССА — Системы линейных уравнений

    Ме́тод Га́усса
    – классический метод решения системы линейных алгебраических уравнений (СЛАУ).
    Это метод последовательного исключения переменных, когда с помощью элементарных
    преобразований система уравнений приводится к равносильной системе треугольного
    вида, из которой последовательно, начиная с последних (по номеру), находятся
    все переменные системы.

    Хотя в настоящее время данный метод повсеместно называется методом Гаусса, он
    был известен и до К. Ф. Гаусса. Первое известное описание данного
    метода – в китайском трактате «Математика в девяти книгах».

    Пусть исходная
    система выглядит следующим образом

    Матрица A называется основной матрицей системы, – столбцом свободных членов.

    Тогда, согласно
    свойству элементарных преобразований над строками, основную матрицу этой
    системы можно привести к ступенчатому виду (эти же преобразования нужно
    применять к столбцу свободных членов):

    При этом будем
    считать, что базисный минор (ненулевой минор максимального порядка) основной матрицы
    находится в верхнем левом углу, то есть в него входят только коэффициенты при
    переменных .

    Тогда переменные называются главными
    переменными
    . Все остальные называются свободными.

    Если хотя бы одно
    число , где , то рассматриваемая система несовместна,
    т.е. у неё нет ни одного решения.

    Пусть для любых .

    Перенесём
    свободные переменные за знаки равенств и поделим каждое из уравнений системы на
    свой коэффициент при самом левом (, где   — номер строки):

    где

    Если свободным
    переменным системы (2) придавать все возможные значения и решать новую систему
    относительно главных неизвестных снизу вверх (то есть от нижнего уравнения к
    верхнему), то мы получим все решения этой СЛАУ. Так как эта система получена
    путём элементарных преобразований над исходной системой (1), то по теореме об
    эквивалентности при элементарных преобразованиях системы (1) и (2)
    эквивалентны, то есть множества их решений совпадают.

    Условие
    совместности

    Упомянутое выше
    условие для всех может быть сформулировано в
    качестве необходимого и достаточного условия совместности:

    Напомним, что
    рангом совместной системы называется ранг её основной матрицы (либо
    расширенной, так как они равны).

    Алгоритм
    решения СЛАУ методом Гаусса подразделяется на два этапа.

    • На первом этапе осуществляется так называемый
      прямой ход, когда путём элементарных преобразований над строками систему
      приводят к ступенчатой или треугольной форме, либо устанавливают, что
      система несовместна. А именно, среди элементов первого столбца матрицы
      выбирают ненулевой, перемещают его на крайнее верхнее положение
      перестановкой строк и вычитают получившуюся после перестановки первую
      строку из остальных строк, домножив её на величину, равную отношению
      первого элемента каждой из этих строк к первому элементу первой строки,
      обнуляя тем самым столбец под ним. После того, как указанные
      преобразования были совершены, первую строку и первый столбец мысленно
      вычёркивают и продолжают пока не останется матрица нулевого размера. Если
      на какой-то из итераций среди элементов первого столбца не нашёлся
      ненулевой, то переходят к следующему столбцу и проделывают аналогичную
      операцию.
    • На втором этапе осуществляется так называемый
      обратный ход, суть которого заключается в том, чтобы выразить все
      получившиеся базисные переменные через небазисные и построить фундаментальную
      систему решений, либо, если все переменные являются базисными, то выразить
      в численном виде единственное решение системы линейных уравнений. Эта
      процедура начинается с последнего уравнения, из которого выражают
      соответствующую базисную переменную (а она там всего одна) и подставляют в
      предыдущие уравнения, и так далее, поднимаясь по «ступенькам» наверх.
      Каждой строчке соответствует ровно одна базисная переменная, поэтому на
      каждом шаге, кроме последнего (самого верхнего), ситуация в точности
      повторяет случай последней строки.

    Метод Гаусса требует арифметических операций.

    В простейшем случае алгоритм выглядит так:

    • Обратный ход. Из последнего ненулевого уравнения
      выражаем базисную переменную через небазисные и подставляем в предыдущие уравнения.
      Повторяя эту процедуру для всех базисных переменных, получаем
      фундаментальное решение.

    Пример 1

    Покажем, как
    методом Гаусса можно решить следующую систему:

    Обнулим
    коэффициенты при во второй и третьей строчках. Для
    этого прибавим к ним первую строчку, умноженную на и 1, соответственно:

    Теперь обнулим
    коэффициент при  в третьей строке, вычтя из неё
    вторую строку, умноженную на 4:

    В результате мы
    привели исходную систему к треугольному виду, тем самым закончим первый этап
    алгоритма.

    На втором этапе
    разрешим полученные уравнения в обратном порядке. Имеем:

    из третьего;

    из второго, подставив
    полученное

    из первого, подставив
    полученные и .

    Таким образом
    исходная система решена.

    В случае, если
    число уравнений в совместной системе получилось меньше числа неизвестных, то
    тогда ответ будет записываться в виде фундаментальной системы решений.

    Матрицы

    и исключение Гаусса

    Назад Замена

    Напомним, что линейная система уравнений состоит из двух или более линейных уравнений с одинаковыми переменными. Линейная система, состоящая из трех уравнений стандартной формы, расположенных так, чтобы переменная x не появлялась ни в одном уравнении после первого, а переменная y не появлялась ни в одном уравнении после второго, называется верхнетреугольной формой. линейная система, состоящая из уравнений с тремя переменными в стандартной форме, расположенная так, что переменная x не появляется после первого уравнения, а переменная y не появляется после второго уравнения.. Например,

    Обратите внимание, что система образует треугольник, в котором каждое последующее уравнение содержит на одну переменную меньше. В целом

    Линейные системы в верхней треугольной форме {a1x + b1y = c1b2y = c2 {a1x + b1y + c1z = d1b2y + c2z = d2c3z = d3

    Если линейная система находится в этой форме, мы можем легко найти одну из переменных, а затем произвести обратную замену, чтобы найти оставшиеся переменные.

    Пример 1

    Решить: {3x − y = 72y = −2.

    Решение:

    Напомним, что решения линейных систем с двумя переменными, если они существуют, представляют собой упорядоченные пары ( x , y ). Мы можем легко определить значение y , используя второе уравнение.

    2y = −2y = −1

    Затем используйте первое уравнение 3x − y = 7 и тот факт, что y = −1, чтобы найти x .

    3x − y = 73x — (- 1) = 73x + 1 = 73x = 6x = 2

    Ответ: (2, −1)

    Пример 2

    Решите: {x − 6y + 2z = 163y − 9z = 5z = −1.

    Решение:

    Напомним, что решения линейных систем с тремя переменными, если они существуют, являются упорядоченными тройками ( x , y , z ). Воспользуйтесь вторым уравнением 3y − 9z = 5 и тем фактом, что z = −1, чтобы найти y .

    3y − 9z = 53y − 9 (−1) = 53y + 9 = 53y = −4y = −43

    Затем подставьте y и z в первое уравнение.

    x − 6y + 2z = 16x − 6 (−43) +2 (−1) = 16x + 8−2 = 16x + 6 = 16x = 10

    Ответ: (10, −43, −1)

    Попробуй! Решите: {4x − y + 3z = 12y − 9z = −23z = 2.

    Ответ: (14, 2, 23)

    Матрицы и исключение Гаусса

    Цель этого раздела — разработать метод, упрощающий процесс решения линейных систем. Мы начинаем с определения матрицы — прямоугольного массива чисел, состоящего из строк и столбцов., Который представляет собой прямоугольный массив чисел, состоящий из строк и столбцов. Для данной линейной системы в стандартной форме мы создаем матрицу коэффициентов Матрицу коэффициентов линейной системы в стандартной форме, записанную так, как они выглядят выстроенной, без переменных или операций.записывая коэффициенты в том виде, в каком они кажутся выстроенными, без переменных или операций, как показано ниже.

    Матрица коэффициентов линейной системы {a1x + b1y + c1z = d1a2x + b2y + c2z = d2a3x + b3y + c3z = d3 ⇒ [a1b1c1a2b2c2a3b3c3]

    Строки представляют коэффициенты в уравнениях, а столбцы представляют коэффициенты каждой переменной. Кроме того, если мы включим столбец, представляющий константы, мы получим так называемую расширенную матрицу — матрицу коэффициентов с включенным столбцом констант.. Для линейной системы с двумя переменными

    Расширенная матрица линейной системы {a1x + b1y = c1a2x + b2y = c2 ⇔ [a1b1 | c1a2b2 | c2]

    А для линейной системы с тремя переменными имеем

    Расширенная матрица линейной системы {a1x + b1y + c1z = d1a2x + b2y + c2z = d2a3x + b3y + c3z = d3 ⇔ [a1b1c1 | d1a2b2c2 | d2a3b3c3 | d3]

    Примечание : Пунктирная вертикальная линия обеспечивает визуальное разделение между матрицей коэффициентов и столбцом констант.В других ресурсах по алгебре, с которыми вы можете столкнуться, это иногда опускается.

    Пример 3

    Постройте расширенную матрицу, которая соответствует: {9x − 6y = 0 − x + 2y = 1.

    Решение:

    Эта система состоит из двух линейных уравнений стандартной формы; следовательно, коэффициенты в матрице отображаются так же, как и в системе.

    {9x − 6y = 0 − x + 2y = 1 ⇔ [9−6 | 0−12 | 1]

    Пример 4

    Постройте расширенную матрицу, которая соответствует: {x + 2y − 4z = 52x + y − 6z = 84x − y − 12z = 13.

    Решение:

    Поскольку уравнения представлены в стандартной форме, коэффициенты появляются в матрице так же, как и в системе.

    {x + 2y − 4z = 52x + y − 6z = 84x − y − 12z = 13 ⇔ [12−4 | 521−6 | 84−1−12 | 13]

    Матрица имеет верхнюю треугольную форму, если все элементы ниже ведущего ненулевого элемента в каждой последующей строке равны нулю. Например,

    Обратите внимание, что элементы ниже главной диагонали равны нулю, а коэффициенты выше образуют треугольную форму.В целом

    Верхняя треугольная форма [a1b10b2] [a1b1c10b2c200c3]

    Это важно, потому что в этом разделе мы очерчиваем процесс, с помощью которого можно выполнить определенные операции для создания эквивалентной линейной системы в верхней треугольной форме, чтобы ее можно было решить с помощью обратной подстановки. Обзор процесса представлен ниже:

    Когда система принимает форму верхнего треугольника, мы можем использовать обратную замену, чтобы легко ее решить.Важно отметить, что представленные здесь расширенные матрицы представляют собой линейные системы уравнений в стандартной форме.

    Следующие элементарные операции со строками Операции, которые могут быть выполнены для получения эквивалентных линейных систем. приводят к расширенным матрицам, которые представляют эквивалентные линейные системы:

    1. Любые две строки можно поменять местами.
    2. Каждый элемент в строке можно умножить на ненулевую константу.
    3. Любая строка может быть заменена суммой этой строки и кратной другой.

    Примечание: Эти операции согласуются со свойствами, используемыми в методе исключения.

    Чтобы эффективно решить систему линейных уравнений, сначала постройте расширенную матрицу. Затем примените соответствующие элементарные операции со строками, чтобы получить расширенную матрицу в форме верхнего треугольника. В этой форме эквивалентная линейная система может быть легко решена с помощью обратной подстановки. Этот процесс называется гауссовским устранением. Шаги, используемые для получения эквивалентной линейной системы в верхней треугольной форме, чтобы ее можно было решить с помощью обратной подстановки., названный в честь Карла Фридриха Гаусса (1777–1855).

    Рисунок 3.1

    Карл Фридрих Гаусс (Википедия)

    Шаги для решения линейного уравнения с двумя переменными с использованием исключения Гаусса перечислены в следующем примере.

    Пример 5

    Решить, используя матрицы и метод исключения Гаусса: {9x − 6y = 0 − x + 2y = 1.

    Решение:

    Перед началом этого процесса убедитесь, что уравнения в системе имеют стандартную форму.

    Шаг 1 : Постройте соответствующую расширенную матрицу.

    {9x − 6y = 0 − x + 2y = 1 ⇔ [9−6 | 0−12 | 1]

    Шаг 2 : Примените операции элементарной строки, чтобы получить верхнюю треугольную форму. В этом случае нам нужно только удалить первый элемент второй строки, −1. Для этого умножьте вторую строку на 9 и прибавьте ее к первой строке.

    Теперь используйте это, чтобы заменить вторую строку.

    [9−6 | 0012 | 9]

    В результате получается расширенная матрица в форме верхнего треугольника.

    Шаг 3 : Преобразуйте обратно в линейную систему и решите, используя обратную подстановку. В этом примере у нас

    [9−6 | 0012 | 9] ⇒ {9x − 6y = 012y = 9

    Решите второе уравнение относительно y ,

    12y = 9y = 912y = 34

    Подставьте это значение вместо y в первое уравнение, чтобы найти x ,

    9x − 6y = 09x − 6 (34) = 09x − 92 = 09x = 92x = 12

    Ответ: (12, 34)

    Шаги по использованию исключения Гаусса для решения линейного уравнения с тремя переменными перечислены в следующем примере.

    Пример 6

    Решить, используя матрицы и метод исключения Гаусса: {x + 2y − 4z = 52x + y − 6z = 84x − y − 12z = 13.

    Решение:

    Перед началом этого процесса убедитесь, что уравнения в системе имеют стандартную форму.

    Шаг 1 : Постройте соответствующую расширенную матрицу.

    {x + 2y − 4z = 52x + y − 6z = 84x − y − 12z = 13 ⇒ [12−4 | 521−6 | 84−1−12 | 13]

    Шаг 2 : Примените операции элементарной строки, чтобы получить верхнюю треугольную форму.Начнем с исключения первого элемента второй строки, в данном случае 2. Для этого умножьте первую строку на −2, а затем добавьте ее во вторую строку.

    [12−4 | 521−6 | 84−1−12 | 13] ⇒ × (−2) −2−48−10 + 21−680−32−2

    Используйте это, чтобы заменить вторую строку.

    [12−4 | 50−32 | −24−1−12 | 13]

    Затем удалите первый элемент третьей строки, в данном случае 4, умножив первую строку на −4 и прибавив ее к третьей строке.

    [12−4 | 50−32 | −24−1−12 | 13] ⇒ × (−4) −4−816−20 + 4−1−12130−94−7

    Используйте это, чтобы заменить третью строку.

    [12−4 | 50−32 | −20−94 | −7]

    Это приводит к расширенной матрице, в которой элементы под первым элементом первой строки равны нулю. Затем удалите второй элемент в третьей строке, в данном случае −9. Умножьте вторую строку на −3 и прибавьте ее к третьей строке.

    Используйте это, чтобы заменить третью строку, и мы видим, что мы получили матрицу в форме верхнего треугольника.

    [12−4 | 50−32 | −200−2 | −1]

    Шаг 3 : Преобразуйте обратно в линейную систему и решите, используя обратную подстановку. В этом примере у нас

    [12−4 | 50−32 | −200−2 | −1] ⇒ {x + 2y − 4z = 5−3y + 2z = −2−2z = −1

    Ответ: Читателю остается убедиться, что решение (5,1,12).

    Примечание: Обычно работа по замене строки путем умножения и сложения выполняется на стороне с использованием бумаги для заметок.

    Пример 7

    Решить, используя матрицы и метод исключения Гаусса: {2x − 9y + 3z = −18x − 2y − 3z = −8−4x + 23y + 12z = 47.

    Решение:

    Начнем с преобразования системы в расширенную матрицу коэффициентов.

    {2x − 9y + 3z = −18x − 2y − 3z = −8−4x + 23y + 12z = 47 ⇒ [2−93 | −181−2−3 | −8−42312 | 47]

    Элементарные операции со строками упрощаются, если ведущий ненулевой элемент в строке равен 1.По этой причине начните с того, что поменяйте местами первый и второй ряды.

    Заменить строку два суммой −2, умноженной на первую и вторую строку.

    Заменить третью строку суммой четырех строк первой и третьей.

    Следующий ряд 3 разделить на 15.

    Поменяйте местами третий ряд со вторым.

    Затем замените строку 3 суммой, умноженной на 5 строк второй и третьей.

    В результате получается матрица в форме верхнего треугольника. Матрица находится в виде эшелона строк Матрица в треугольной форме, где ведущий ненулевой элемент каждой строки равен 1., если она находится в верхней треугольной форме, где ведущий ненулевой элемент каждой строки равен 1. Мы можем получить эту форму, заменив третью строку на результат деления на 9.

    Преобразуйте в систему линейных уравнений и решите обратной подстановкой.

    [1−2−3 | −8010 | 1001 | 13] ⇒ {x − 2y − 3z = −8y = 1z = 13

    Здесь y = 1 и z = 13. Подставляем в первое уравнение, чтобы найти x .

    x − 2y − 3y = −8x − 2 (1) −3 (13) = — 8x − 2−1 = −8x − 3 = −8x = −5

    Ответ: Следовательно, решение (−5, 1, 13).

    Технологическое примечание : Многие современные калькуляторы и системы компьютерной алгебры могут выполнять метод исключения Гаусса. Сначала вам нужно узнать, как войти в матрицу.Затем используйте функции калькулятора, чтобы найти форму эшелона строки. Предлагаем вам провести исследование по этой теме для вашей конкретной модели калькулятора.

    Попробуй! Решите, используя метод исключения Гаусса: {x − 3y + 2z = 164x − 11y − z = 692x − 5y − 4z = 36.

    Ответ: (6, −4, −1)

    Напомним, что некоторые непротиворечивые линейные системы зависимы, то есть у них бесконечно много решений.А некоторые линейные системы не имеют одновременного решения; это несовместимые системы.

    Пример 8

    Решить, используя матрицы и метод исключения Гаусса: {x − 2y + z = 42x − 3y + 4z = 74x − 7y + 6z = 15.

    Решение:

    Начнем с преобразования системы в расширенную матрицу коэффициентов.

    {x − 2y + z = 42x − 3y + 4z = 74x − 7y + 6z = 15 ⇒ [1−21 | 42−34 | 74−76 | 15]

    Заменить вторую строку на −2 (строка 1) + (строка 2) и заменить строку три на −4 (строка 1) + (строка 3).

    [1−21 | 4012 | −1012 | −1]

    Заменить третью строку на −1 (строка 2) + (строка 3).

    [1-21 | 4012 | -1000 | 0]

    Последняя строка указывает, что это зависимая система, потому что преобразование расширенной матрицы обратно в уравнения, которые у нас есть,

    {x − 2y + z = 4y + 2z = −10x + 0y + 0z = 0

    Обратите внимание, что строка нулей соответствует следующему идентификатору:

    0x + 0y + 0z = 00 = 0 ✓

    В этом случае мы можем выразить бесконечно много решений через z .Из второго ряда имеем:

    y + 2z = −1y = −2z − 1

    И из первого уравнения

    x − 2y + z = 4x − 2 (−2z − 1) + z = 4x + 5z + 2 = 4x = −5z + 2

    Решения имеют вид (x, y, z) = (- 5z + 2, −2z − 1, z), где z — любое действительное число.

    Ответ: (−5z + 2, −2z − 1, z)

    Зависимые и несовместимые системы могут быть идентифицированы в расширенной матрице коэффициентов, когда все коэффициенты в одной строке равны нулю.

    Если строка нулей имеет соответствующую константу, равную нулю, тогда матрица представляет зависимую систему. Если константа отлична от нуля, матрица представляет собой несовместимую систему.

    Попробуй! Решите, используя матрицы и метод исключения Гаусса: {5x − 2y + z = −310x − y + 3z = 0−15x + 9y − 2z = 17.

    Ответ: Ø

    Основные выводы

    • Линейная система в верхней треугольной форме может быть легко решена с помощью обратной подстановки.
    • Расширенная матрица коэффициентов и метод исключения Гаусса могут использоваться для упрощения процесса решения линейных систем.
    • Чтобы решить систему с использованием матриц и исключения Гаусса, сначала используйте коэффициенты для создания расширенной матрицы. Примените операции с элементарными строками как средство для получения матрицы в форме верхнего треугольника. Преобразуйте матрицу обратно в эквивалентную линейную систему и решите ее, используя обратную подстановку.

    Тематические упражнения

      Часть A: Обратная замена

        Решите, используя обратную замену.

      1. {5x − 3y = 2y = −1

      2. {3x + 2y = 1y = 3

      3. {x − 4y = 12y = −3

      4. {x − 5y = 310y = −6

      5. {4x − 3y = −167y = 0

      6. {3x − 5y = −104y = 8

      7. {2x + 3y = −13y = 2

      8. {6x − y = −34y = 3

      9. {х-у = 02у = 0

      10. {2x + y = 23y = 0

      11. {x + 3y − 4z = 1y − 3z = −2z = 3

      12. {x − 5y + 4z = −1y − 7z = 10z = −2

      13. {x − 6y + 8z = 23y − 4z = −42z = −1

      14. {2x − y + 3z = −92y + 6z = −23z = 2

      15. {10x − 3y + z = 1311y − 3z = 92z = −6

      16. {3x − 2y + 5z = −244y + 5z = 34z = −12

      17. {x − y + 2z = 12y + z = 13z = −1

      18. {x + 2y − z = 2y − 3z = 16z = 1

      19. {x − 9y + 5z = −32y = 103z = 27

      20. {4x — z = 33y − 2z = −12z = −8

      Часть B: Матрицы и исключение Гаусса

        Построить соответствующую расширенную матрицу (не решать).

      1. {х + 2у = 34х + 5у = ​​6

      2. {6x + 5y = 43x + 2y = 1

      3. {x − 2y = 12x − y = 1

      4. {х-у = 2-х + у = -1

      5. {−x + 8y = 32y = 2

      6. {3x − 2y = 4 − y = 5

      7. {3x − 2y + 7z = 84x − 5y − 10z = 6 − x − 3y + 2z = −1

      8. {x − y − z = 02x − y + 3z = −1 − x + 4y − 3z = −2

      9. {x − 9y + 5z = −32y = 103z = 27

      10. {4x − z = 33y − 2z = −12z = −8

      11. {8x + 2y = −13−2y + z = 112x − 5z = −18

      12. {x − 3z = 2y + 6z = 42x + 3y = 12

        Решите, используя матрицы и метод исключения Гаусса.

      1. {x − 5y = 22x − y = 1

      2. {x − 2y = −1x + y = 1

      3. {10x − 7y = 15−2x + 3y = −3

      4. {9x − 10y = 23x + 5y = −1

      5. {3x + 5y = 82x − 3y = 18

      6. {5x − 3y = −147x + 2y = −1

      7. {9x + 15y = 53x + 5y = 7

      8. {6x − 8y = 1−3x + 4y = −1

      9. {х + у = 0х-у = 0

      10. {7x − 3y = 03x − 7y = 0

      11. {2x − 3y = 4−10x + 15y = −20

      12. {6x − 10y = 20−3x + 5y = −10

      13. {x + y − 2z = −1 − x + 2y − z = 1x − y + z = 2

      14. {x − y + z = −2x + 2y − z = 6 − x + y − 2z = 3

      15. {2x − y + z = 2x − y + z = 2−2x + 2y − z = −1

      16. {3x − y + 2z = 7 − x + 2y + z = 6x + 3y − 2z = 1

      17. {x − 3y + z = 6 − x − y + 2z = 42x + y + z = 3

      18. {4x − y + 2z = 12x − 3y + 2z = 7−2x + 3y + 4z = −16

      19. {2x − 4y + 6z = −43x − 2y + 5z = −25x − y + 2z = 1

      20. {3x + 6y + 9z = 62x − 2y + 3z = 0−3x + 18y − 12z = 5

      21. {−x + y − z = −23x − 2y + 5z = 13x − 5y − z = 3

      22. {x + 2y + 3z = 43x + 8y + 13z = 212x + 5y + 8z = 16

      23. {2x − 4y − 5z = 3 − x + y + z = 13x − 4y − 5z = −4

      24. {5x − 3y − 2z = 43x − 6y + 4z = −6 − x + 2y − z = 2

      25. {−2x − 3y + 12z = 44x − 5y − 10z = −1 − x − 3y + 2z = 0

      26. {3x − 2y + 5z = 104x + 3y − 3z = −6x + y + z = 2

      27. {x + 2y + z = −3x + 6y + 3z = 7x + 4y + 2z = 2

      28. {2x − y + z = 14x − y + 3z = 52x + y + 3z = 7

      29. {2x + 3y − 4z = 03x − 5y + 3z = −105x − 2y + 5z = −4

      30. {3x − 2y + 9z = 2−2x − 5y − 4z = 35x − 3y + 3z = 15

      31. {8x + 2y = −13−2y + z = 112x − 5z = −18

      32. {x − 3z = 2y + 6z = 42x + 3y = 12

      33. {9x + 3y − 11z = 62x + y − 3z = 17x + 2y − 8z = 3

      34. {3x − y − z = 4−5x + y + 2z = −36x − 2y − 2z = 8

      35. {2x − 4y + 3z = 153x − 5y + 2z = 185x + 2y − 6z = 0

      36. {3x − 4y − 3z = −144x + 2y + 5z = 12−5x + 8y − 4z = −3

      Часть C: Обсуждение

      1. Изучите и обсудите историю метода исключения Гаусса.Кто первым разработал этот процесс? Опубликуйте что-нибудь, что вам показалось интересным из этой истории.

      2. Изучите и обсудите историю современной матричной записи. Кому засчитывается разработка? В каких сферах они используются сегодня? Разместите свои выводы на доске обсуждений.

    ответов

    1. (-15, -1)

    2. (-5, -32)

    3. (-32,23)

    4. (−6, −2, −12)

    5. (85,0, −3)

    6. (73,23, −13)

    1. [12 | 345 | 6]

    2. [1-2 | 12-1 | 1]

    3. [−18 | 302 | 2]

    4. [3−27 | 84−5−10 | 6−1−32 | −1]

    5. [1−95 | −3020 | 10003 | 27]

    6. [820 | −130−21 | 1120−5 | −18]

    7. (13, −13)

    8. (32,0)

    9. (х, 23x − 43)

    10. (12,12, −12)

    11. (1,0,12)

    12. (−8, −12z + 52, z)

    13. (-32, -12, 0)

    Решающих систем с исключением Гаусса · Precalculus

    Решение систем с исключением Гаусса · Предварительное вычисление

    В этом разделе вы:

    • Напишите расширенную матрицу системы уравнений.
    • Напишите систему уравнений из расширенной матрицы.
    • Выполнение операций со строками в матрице.
    • Решите систему линейных уравнений с помощью матриц.

    Карл Фридрих Гаусс жил в конце 18-го и начале 19-го веков, но до сих пор считается одним из самых плодовитых математиков в истории. Его вклад в математику и физику охватывает такие области, как алгебра, теория чисел, анализ, дифференциальная геометрия, астрономия и оптика.Его открытия в области теории матриц изменили способ работы математиков за последние два столетия.

    Мы впервые столкнулись с исключением Гаусса в системах линейных уравнений: две переменные. В этом разделе мы еще раз вернемся к этой технике решения систем, на этот раз с использованием матриц.

    Написание расширенной матрицы системы уравнений

    Матрица может служить устройством для представления и решения системы уравнений. Чтобы выразить систему в матричной форме, мы извлекаем коэффициенты переменных и констант, и они становятся элементами матрицы.Мы используем вертикальную линию, чтобы отделить записи коэффициентов от констант, по сути заменяя знаки равенства. Когда система написана в такой форме, мы называем ее расширенной матрицей .

    Например, рассмотрим следующий 2 × 2

    система уравнений.

    3x + 4y = 74x − 2y = 5

    Мы можем записать эту систему в виде расширенной матрицы:

    [344−2 \ | 75]

    Мы также можем написать матрицу, содержащую только коэффициенты. Это называется матрицей коэффициентов .

    [344−2]

    Трехкратная система уравнений , например

    3x − y − z = 0 x + y = 5 2x − 3z = 2

    имеет матрицу коэффициентов

    [3−1−111020−3]

    и представлена ​​расширенной матрицей

    [3−1−111020−3 \ | 052]

    Обратите внимание, что матрица написана так, что переменные выстраиваются в свои собственные столбцы: x -термов идет в первый столбец, -термов — во втором столбце и z -термов — в третьем столбце.Очень важно, чтобы каждое уравнение было записано в стандартной форме ax + by + cz = d

    , чтобы переменные совпадали. Если в уравнении отсутствует переменный член, коэффициент равен 0.

    Для данной системы уравнений напишите расширенную матрицу.

    1. Запишите коэффициенты членов x как числа в первом столбце.
    2. Запишите коэффициенты членов и в виде чисел во втором столбце.
    3. Если имеется z -термов, запишите коэффициенты как числа в третьем столбце.
    4. Нарисуйте вертикальную линию и напишите константы справа от нее.

    Написание расширенной матрицы для системы уравнений

    Напишите расширенную матрицу для данной системы уравнений.

    x + 2y − z = 3 2x − y + 2z = 6 x − 3y + 3z = 4

    Расширенная матрица отображает коэффициенты переменных и дополнительный столбец для констант.

    [12−12−121−33 \ | 364]

    Запишите расширенную матрицу данной системы уравнений.

    4x − 3y = 113x + 2y = 4

    [4−33 2 \ | 11 4]

    Написание системы уравнений из расширенной матрицы

    Мы можем использовать расширенные матрицы, чтобы помочь нам решать системы уравнений, потому что они упрощают операции, когда системы не обременены переменными. Однако важно понимать, как переключаться между форматами, чтобы поиск решений был более плавным и интуитивно понятным.Здесь мы будем использовать информацию в расширенной матрице, чтобы записать систему уравнений в стандартной форме.

    Написание системы уравнений из расширенной матричной формы

    Найдите систему уравнений из расширенной матрицы.

    [1−3−52−5−4−354 \ | -256]

    Когда столбцы представляют переменные x,

    y,

    и z,

    [1−3−52−5−4−354 \ | −256] → x − 3y − 5z = −2 2x − 5y − 4z = 5−3x + 5y + 4z = 6

    Напишите систему уравнений из расширенной матрицы.

    [1−1 12−1 30 1 1 \ | 5 1-9]

    x — y + z = 52x — y + 3z = 1 y + z = −9

    Выполнение операций со строками в матрице

    Теперь, когда мы можем писать системы уравнений в форме расширенной матрицы, мы рассмотрим различные операции со строками , которые могут выполняться с матрицей, такие как сложение, умножение на константу и перестановка строк.

    Выполнение строковых операций над матрицей — это метод, который мы используем для решения системы уравнений.Чтобы решить систему уравнений, мы хотим преобразовать матрицу в форму строки-эшелона , в которой есть единицы по главной диагонали от верхнего левого угла до нижнего правого угла и нули в каждой позиции. ниже главной диагонали, как показано.

    Рядно-эшелонированная форма [1ab01d001]

    Мы используем операции со строками, соответствующие операциям с уравнениями, чтобы получить новую матрицу, которая эквивалентна строкам в более простой форме. Вот рекомендации по получению формы рядного эшелона.

    1. В любой ненулевой строке первым ненулевым числом является 1. Оно называется ведущим 1.
    2. Любые нулевые строки помещаются внизу матрицы.
    3. Любая ведущая 1 находится ниже и правее предыдущей ведущей 1.
    4. Любой столбец, в начале которого стоит 1, имеет нули во всех остальных позициях в столбце.

    Чтобы решить систему уравнений, мы можем выполнить следующие операции со строками, чтобы преобразовать матрицу коэффициентов в строковую форму и выполнить обратную подстановку, чтобы найти решение.

    1. Поменять местами ряды. (Обозначение:
      Ri ↔ Rj

      )

    2. Умножьте строку на константу. (Обозначение:
      cRi

      )

    3. Добавить произведение одной строки на константу к другой строке. (Обозначение:
      Ri + cRj)

    Каждая из строковых операций соответствует операциям, которые мы уже научились решать системы уравнений с тремя переменными. С помощью этих операций есть несколько ключевых ходов, которые быстро достигнут цели написания матрицы в виде эшелона строк.Чтобы получить матрицу в виде эшелона строк для поиска решений, мы используем метод исключения Гаусса, который использует операции со строками для получения 1 в качестве первой записи, так что строку 1 можно использовать для преобразования оставшихся строк.

    Исключение по Гауссу

    Метод исключения Гаусса относится к стратегии, используемой для получения матрицы в виде строки-эшелона. Цель — написать матрицу A

    с цифрой 1 в начале главной диагонали и всеми нулями внизу.

    A = [a11a12a13a21a22a23a31a32a33] → После исключения Гаусса A = [1 b12 b130 1 b230 0 1]

    Первый шаг стратегии Гаусса включает получение 1 в качестве первой записи, так что строка 1 может использоваться для изменения строк ниже.

    Учитывая расширенную матрицу, выполните операции со строками для получения формы «строка-эшелон».

    1. Первое уравнение должно иметь старший коэффициент 1. При необходимости поменяйте местами строки или умножьте на константу.
    2. Используйте операции со строками, чтобы получить нули в первом столбце под первой записью 1.
    3. Используйте операции со строками, чтобы получить 1 в строке 2, столбце 2.
    4. Используйте операции со строками, чтобы получить нули в столбце 2 под записью 1.
    5. Используйте операции со строками, чтобы получить 1 в строке 3, столбец 3.
    6. Продолжайте этот процесс для всех строк, пока в каждой записи по главной диагонали не будет 1, а внизу будут только нули.
    7. Если какие-либо строки содержат все нули, поместите их внизу.

    Решение
    2 × 2
    Система методом исключения Гаусса

    Решите данную систему методом исключения Гаусса.

    2x + 3y = 6 x − y = 12

    Сначала мы запишем это как расширенную матрицу.

    [231−1 \ | 612]

    Нам нужна 1 в строке 1, столбце 1. Этого можно добиться, поменяв местами строку 1 и строку 2.

    R1↔R2 → [1−123 \ | 126]

    Теперь у нас есть 1 как первая запись в строке 1, столбце 1.Теперь давайте получим 0 в строке 2, столбце 1. Этого можно добиться, умножив строку 1 на −2,

    .

    , а затем добавляем результат в строку 2.

    −2R1 + R2 = R2 → [1−105 \ | 125]

    У нас есть только один шаг, чтобы умножить строку 2 на 15.

    15R2 = R2 → [1−101 \ | 121]

    Использовать обратную замену. Вторая строка матрицы представляет y = 1.

    Обратная замена y = 1

    в первое уравнение.

    х− (1) = 12 х = 32

    Решением является точка (32,1).

    Решите данную систему методом исключения Гаусса.

    4x + 3y = 11 x − 3y = −1

    (2, 1)

    Использование исключения Гаусса для решения системы уравнений

    Используйте исключение Гаусса для решения заданного 2 × 2

    Система уравнений .

    2x + y = 14x + 2y = 6

    Запишите систему как расширенную матрицу .

    [2142 \ | 16]

    Получить 1 в строке 1, столбце 1.Это можно сделать, умножив первую строку на 12.

    12R1 = R1 → [11242 \ | 126] ​​

    Далее нам нужен 0 в строке 2, столбце 1. Умножим строку 1 на −4

    .

    и добавьте строку 1 к строке 2.

    −4R1 + R2 = R2 → [11200 \ | 124]

    Вторая строка представляет уравнение 0 = 4.

    Следовательно, система непоследовательна и не имеет решения.

    Решение зависимой системы

    Решите систему уравнений.

    3x + 4y = 126x + 8y = 24

    Выполните операций со строками над расширенной матрицей, чтобы попытаться получить строковую форму .

    A = [3468 \ | 1224]

    −12R2 + R1 = R1 → [0068 \ | 024] R1↔R2 → [6800 \ | 24 0]

    Матрица заканчивается всеми нулями в последней строке: 0y = 0.

    Таким образом, существует бесконечное количество решений и система классифицируется как зависимая. Чтобы найти общее решение, вернитесь к одному из исходных уравнений и решите относительно y.

    3x + 4y = 12 4y = 12−3x y = 3−34x

    Итак, решение этой системы — (x, 3−34x).

    Выполнение операций со строками в расширенной матрице 3 × 3 для получения формы эшелона строк

    Выполняет строковые операции с заданной матрицей для получения формы «строка-эшелон».

    [1−342−56−334 \ | 366]

    В первой строке уже есть 1 в строке 1, столбце 1. Следующий шаг — умножить строку 1 на −2

    и добавьте его во 2 строку.Затем замените строку 2 результатом.

    −2R1 + R2 = R2 → [1−3401−2−334 \ | 306]

    Затем получить ноль в строке 3, столбце 1.

    3R1 + R3 = R3 → [1−3401−20−616 \ | 3015]

    Затем получить ноль в строке 3, столбце 2.

    6R2 + R3 = R3 → [1−3401−2004 \ | 3015]

    Последний шаг — получить 1 в строке 3, столбец 3.

    14R3 = R3 → [1−3401−2001 \ | 3−6154]

    Запишите систему уравнений в виде ряда.

    x − 2y + 3z = 9 −x + 3y = −42x − 5y + 5z = 17

    [1−52 52 0 15 0 0 1 \ | 17292]

    Решение системы линейных уравнений с использованием матриц

    Мы видели, как написать систему уравнений с расширенной матрицей , а затем как использовать строковые операции и обратную подстановку для получения эшелонированной формы . Теперь мы перейдем на шаг дальше от строковой формы, чтобы решить систему линейных уравнений 3 на 3.Общая идея состоит в том, чтобы исключить все переменные, кроме одной, с помощью операций со строками, а затем выполнить обратную замену для поиска других переменных.

    Решение системы линейных уравнений с помощью матриц

    Решите систему линейных уравнений с помощью матриц.

    x — y + z = 82x + 3y — z = −23x — 2y −9z = 9

    Сначала мы пишем расширенную матрицу.

    [1−1123−13−2−9 \ | 8−29]

    Затем мы выполняем строковые операции, чтобы получить форму «строка-эшелон».

    −2R1 + R2 = R2 → [1−1105−33−2−9 \ | 8−189] −3R1 + R3 = R3 → [1−1105−301−12 \ | 8−18−15]

    Самый простой способ получить 1 в строке 2 столбца 1 — это поменять местами R2

    и R3.

    Развязки R2 и R3 → [1−11801−12−1505−3−18]

    Затем

    −5R2 + R3 = R3 → [1−1101−120057 \ | 8−1557] −157R3 = R3 → [1−1101−12001 \ | 8−151]

    Последняя матрица представляет собой эквивалентную систему.

    x − y + z = 8 y − 12z = −15 z = 1

    Используя обратную подстановку, мы получаем решение как (4, −3,1).

    Решение зависимой системы линейных уравнений с помощью матриц

    Решите следующую систему линейных уравнений, используя матрицы.

    −x − 2y + z = −1 2x + 3y = 2 y − 2z = 0

    Запишите расширенную матрицу.

    [−1−2123001−2 \ | -120]

    Сначала умножаем строку 1 на −1

    , чтобы получить 1 в строке 1, столбце 1. Затем выполните операций со строками для получения формы «строка-эшелон».

    −R1 → [12−123001−2 \ | 120]

    R2↔R3 → [12−101−2230 \ | 102]

    −2R1 + R3 = R3 → [12−101−20−12 \ | 100]

    R2 + R3 = R3 → [12−101−2000 \ | 210]

    Последняя матрица представляет следующую систему.

    x + 2y − z = 1 y − 2z = 0 0 = 0

    Мы видим по тождеству 0 = 0

    , что это зависимая система с бесконечным числом решений. Затем мы находим общее решение.Решив второе уравнение относительно y

    и подставив его в первое уравнение, мы можем решить для z

    в пересчете на х.

    x + 2y − z = 1 y = 2zx + 2 (2z) −z = 1 x + 3z = 1 z = 1 − x3

    Теперь подставим выражение для z

    во второе уравнение, чтобы найти y

    в пересчете на х.

    y − 2z = 0 z = 1 − x3 y − 2 (1 − x3) = 0 y = 2−2×3

    Общее решение — (x, 2−2×3,1 − x3).

    Решите систему, используя матрицы.

    х + 4y − z = 42x + 5y + 8z = 15x + 3y − 3z = 1

    (1, 1, 1)

    Можно ли решить любую систему линейных уравнений методом исключения Гаусса?

    Да, система линейных уравнений любого размера может быть решена методом исключения Гаусса.

    Дана система уравнений, решите с помощью матриц с помощью калькулятора.

    1. Сохранить расширенную матрицу как матричную переменную
      [A], [B], [C],….
    2. Используйте функцию ref ( в калькуляторе, вызывая каждую матричную переменную по мере необходимости.

    Решение систем уравнений с матрицами с помощью калькулятора

    Решите систему уравнений.

    5x + 3y + 9z = −1−2x + 3y − z = −2 − x − 4y + 5z = 1

    Напишите расширенную матрицу для системы уравнений.

    [539−23−1−1−45 \ | −1−2−1]

    На странице матриц калькулятора введите расширенную матрицу выше в качестве матричной переменной [A].

    [A] = [539−1−23−1−2−1−451]

    Используйте в калькуляторе функцию ref (, вызывая матричную переменную [A].

    исх. ([A])

    Оценить.

    [1 35 95150 1 1321−470 0 1−24187] → x + 35y + 95z = −15 y + 1321z = −47 z = −24187

    При использовании обратной подстановки решение: (61187, −

    , −24187).

    Применение матриц 2 × 2 к финансам

    Кэролайн инвестирует в общей сложности 12 000 долларов в две муниципальные облигации, одна из которых выплачивает 10.5% годовых, а другой — 12% годовых. Годовой процент, полученный по двум инвестициям в прошлом году, составил 1335 долларов. Сколько было вложено по каждой ставке?

    У нас есть система двух уравнений с двумя переменными. Пусть x =

    — сумма, вложенная под 10,5% годовых, а y =

    сумма вложена под 12% годовых.

    x + y = 12,0000,105x + 0,12y = 1,335

    В качестве матрицы имеем

    [110.1050.12 \ | 12,0001,335]

    Умножить строку 1 на −0,105

    и добавьте результат в строку 2.

    [1100.015 \ | 12,00075]

    Затем,

    0,015y = 75 y = 5,000

    Итак, 12 000−5 000 = 7 000.

    Таким образом, 5000 долларов были инвестированы под 12% и 7000 долларов под 10,5%.

    Применение матриц 3 × 3 к финансам

    Ava инвестирует в общей сложности 10 000 долларов в три счета, один из которых платит 5% годовых, другой — 8%, а третий — 9%.Годовой процент, полученный по трем инвестициям в прошлом году, составил 770 долларов. Сумма, вложенная под 9%, была вдвое больше, чем сумма, вложенная под 5%. Сколько было вложено по каждой ставке?

    У нас есть система трех уравнений с тремя переменными. Пусть х

    будет сумма, вложенная под 5% годовых, пусть y

    будет сумма, вложенная под 8% годовых, и пусть z

    — сумма, вложенная под 9% годовых. Таким образом,

    х + у + г = 10,0000.05x + 0,08y + 0,09z = 770 2x − z = 0

    В качестве матрицы имеем

    [1110.050.080.0920−1 \ | 10,0007700]

    Теперь мы выполняем исключение Гаусса, чтобы получить форму строки-эшелон.

    −0.05R1 + R2 = R2 → [11100.030.0420−1 \ | 10,0002700] −2R1 + R3 = R3 → [11100.030.040−2−3 \ | 10,000270−20,000] 10.03R2 = R2 → [01101430 −2−3 \ | 10,0009,000−20,000] 2R2 + R3 = R3 → [111014300−13 \ | 10,0009,000-2,000]

    Третья строка говорит нам -13z = -2,000;

    , таким образом, z = 6000.

    Вторая строка говорит нам, что y + 43z = 9000.

    Подставляем z = 6,000,

    получаем

    y + 43 (6000) = 9000 лет + 8000 = 9000 лет = 1000

    Первая строка сообщает нам, что x + y + z = 10,000.

    Подставляем y = 1,000

    и z = 6000,

    получаем

    х + 1,000 + 6,000 = 10,000 х = 3,000

    Ответ: 3000 долларов вложены под 5%, 1000 долларов вложены под 8% и 6000 долларов вложены под 9%.

    Небольшая обувная компания взяла ссуду в размере 1 500 000 долларов на расширение своего ассортимента. Часть денег была взята под 7%, часть — под 8%, часть — под 10%. Сумма займа под 10% в четыре раза превышала сумму займа под 7%, а годовая процентная ставка по всем трем займам составляла 130 500 долларов. Используйте матрицы, чтобы найти сумму займа по каждой ставке.

    150 000 долларов США под 7%, 750 000 долларов США под 8%, 600 000 долларов США под 10%

    Ключевые понятия

    • Расширенная матрица — это матрица, которая содержит коэффициенты и константы системы уравнений.См. [Ссылка].
    • Матрица, дополненная постоянным столбцом, может быть представлена ​​как исходная система уравнений. См. [Ссылка].
    • Операции со строками включают в себя умножение строки на константу, добавление одной строки к другой строке и замену строк местами.
    • Мы можем использовать метод исключения Гаусса для решения системы уравнений. См. [Ссылка], [ссылка] и [ссылка].
    • Операции со строками выполняются над матрицами для получения формы «строка-эшелон». См. [Ссылка].
    • Чтобы решить систему уравнений, запишите ее в форме расширенной матрицы.Выполните операции со строками, чтобы получить форму эшелона строк. Обратный заменитель, чтобы найти решения. См. [Ссылка] и [ссылка].
    • Калькулятор можно использовать для решения систем уравнений с использованием матриц. См. [Ссылка].
    • Многие реальные проблемы можно решить с помощью расширенных матриц. См. [Ссылка] и [ссылка].

    Упражнения по разделам

    Устный

    Можно ли записать любую систему линейных уравнений в виде расширенной матрицы? Объясните, почему да или почему нет. Объясните, как написать эту расширенную матрицу.

    Да. Для каждой строки коэффициенты переменных записываются поперек соответствующей строки и помещается вертикальная черта; затем константы помещаются справа от вертикальной полосы.

    Можно ли любую матрицу записать в виде системы линейных уравнений? Объясните, почему да или почему нет. Объясните, как написать эту систему уравнений.

    Есть только один правильный метод использования операций со строками в матрице? Попытайтесь объяснить две различные операции со строками, которые могут быть использованы для решения расширенной матрицы [931−2 \ | 06].

    Нет, существует множество правильных методов использования строковых операций над матрицей. Возможны два способа: (1) Поменять местами строки 1 и 2. Тогда R2 = R2−9R1.

    (2) R2 = R1−9R2.

    Затем разделите строку 1 на 9.

    Можно ли решить матрицу с нулевым элементом на диагонали? Объясните, почему да или почему нет. Что бы вы сделали, чтобы исправить ситуацию?

    Может ли матрица с 0 элементами для всей строки иметь одно решение? Объясните, почему да или почему нет.

    Нет. Матрица с 0 элементами для всей строки будет иметь либо ноль, либо бесконечно много решений.

    Алгебраические

    Для следующих упражнений напишите расширенную матрицу линейной системы.

    16y = 49x − y = 2

    [0169−1 \ | 42]

    3x + 2y + 10z = 3−6x + 2y + 5z = 13 4x + z = 18

    x + 5y + 8z = 19 12x + 3y = 43x + 4y + 9z = −7

    [1581230349 \ | 164-7]

    6x + 12y + 16z = 4 19x − 5y + 3z = −9 x + 2y = −8

    Для следующих упражнений запишите линейную систему из расширенной матрицы.

    [−256−18 \ | 526]

    −2x + 5y = 56x − 18y = 26

    [320−1−94857 \ | 3−18]

    3x + 2y = 13 − x − 9y + 4z = 538x + 5y + 7z = 80

    [8291−175003 \ | 433810]

    [45−2015887−3 \ | 122-5]

    4x + 5y − 2z = 12 y + 58z = 28x + 7y − 3z = −5

    Для следующих упражнений решите систему методом исключения Гаусса.

    [−124−5 \ | −36]

    (-1, -2)

    2x − 3y = −95x + 4y = 58

    (6,7)

    2x + 3y = 12 4x + y = 14

    (3,2)

    −4x − 3y = −2 3x − 5y = −13

    −5x + 8y = 3 · 10x + 6y = 5

    (15,12)

    3x + 4y = 12−6x − 8y = −24

    −60x + 45y = 12 20x − 15y = −4

    (х, 415 (5х + 1))

    −1.06x − 2,25y = 5,51−5,03x − 1,08y = 5,40

    34x − 35y = 414x + 23y = 1

    (19639, −513)

    [100011001 \ | 314587]

    (31, -42,87)

    [101110011 \ | 5020−90]

    [123056008 \ | 479]

    (2140,120,98)

    [−0.10.3−0.1−0.40.20.10.60.10.7 \ | 0,20,8−0,8]

    −2x + 3y − 2z = 3 4x + 2y − z = 9 4x − 8y + 2z = −6

    (1813,1513, −1513)

    x + y − 4z = −4 5x − 3y − 2z = 0 2x + 6y + 7z = 30

    2x + 3y + 2z = 1 −4x − 6y − 4z = −2 10x + 15y + 10z = 5

    (х, у, 12 (1−2x − 3y))

    x + 2y − z = 1 − x − 2y + 2z = −23x + 6y − 3z = 5

    x + 2y − z = 1 − x − 2y + 2z = −2 3x + 6y − 3z = 3

    (х, −x2, −1)

    Х + у = 2 х + г = 1-у-г = -3

    x + y + z = 100 x + 2z = 125 − y + 2z = 25

    (125, −25,0)

    14x − 23z = −1215x + 13y = 4715y − 13z = 29

    −12x + 12y + 17z = −5314 12x − 12y + 14z = 3 14x + 15y + 13z = 2315

    (8,1, −2)

    −12x − 13y + 14z = −296 15x + 16y − 17z = 431210−18x + 19y + 110z = −4945

    Расширения

    Для следующих упражнений используйте метод исключения Гаусса для решения системы.

    х-17 + у-28 + г-34 = 0 х + у + г = 6 х + 23 + 2у + г-33 = 5

    (1,2,3)

    x − 14 − y + 14 + 3z = −1 x + 52 + y + 74 − z = 4 x + y − z − 22 = 1

    x − 34 − y − 13 + 2z = −1x + 52 + y + 52 + z + 52 = 8 x + y + z = 1

    (х, 3128−3×4,128 (−7x − 3))

    x − 310 + y + 32−2z = 3 x + 54 − y − 18 + z = 32x − 14 + y + 42 + 3z = 32

    x − 34 − y − 13 + 2z = −1x + 52 + y + 52 + z + 52 = 7 x + y + z = 1

    Реальные приложения

    Для следующих упражнений настройте расширенную матрицу, описывающую ситуацию, и найдите желаемое решение.

    Ежедневно в магазине кексов продается 5 000 кексов со вкусом шоколада и ванили. Если вкус шоколада в 3 раза популярнее, чем аромат ванили, сколько кексов продается в день?

    В конкурирующем магазине кексов ежедневно продаются кексы на 4520 долларов. Шоколадные кексы стоят 2,25 доллара, а кексы из красного бархата — 1,75 доллара. Если общее количество кексов, проданных в день, составляет 2200, сколько штук каждого вкуса продается каждый день?

    860 красный бархат, 1340 шоколадный

    Вы вложили 10 000 долларов в два счета: один с простой процентной ставкой 3%, другой — с 2.5% годовых. Если ваша общая выплата процентов по истечении одного года составила 283,50 доллара, какая сумма была на каждом счете по истечении года?

    Вы инвестировали 2300 долларов на счет 1 и 2700 долларов на счет 2. Если общая сумма процентов по истечении одного года составляет 254 доллара, а на счете 2 процентная ставка в 1,5 раза выше, чем на счете 1, каковы процентные ставки? Предположим простые процентные ставки.

    4% на счет 1, 6% на счет 2

    Bikes’R’Us производит велосипеды по 250 долларов.Он стоит производителю 180 долларов за велосипед плюс стартовый взнос в размере 3500 долларов. Через сколько проданных велосипедов производитель выйдет на уровень безубыточности?

    Крупный магазин бытовой техники рассматривает возможность приобретения пылесосов у небольшого производителя. Магазин сможет приобрести пылесосы по 86 долларов каждый, со стоимостью доставки 9 200 долларов, независимо от того, сколько пылесосов будет продано. Если магазин должен начать получать прибыль после продажи 230 единиц, сколько они должны взимать за пылесосы?

    Три самых популярных вкуса мороженого — это шоколад, клубника и ваниль, составляющие 83% вкусов, продаваемых в магазине мороженого.Если ваниль продается на 1% больше, чем в два раза больше клубники, а шоколад продается на 11% больше, чем ваниль, сколько в общем потреблении мороженого приходится на ароматы ванили, шоколада и клубники?

    В магазине мороженого растет спрос на три вкуса. В прошлом году банановое, тыквенное и мороженое с каменистой дорогой составили 12% от общего объема продаж мороженого. В этом году на те же три вида мороженого пришлось 16,9% продаж мороженого. Продажи по каменистой дороге увеличились вдвое, продажи бананов увеличились на 50%, а продажи тыквы увеличились на 20%.Если у мороженого по каменистой дороге было на один процент меньше продаж, чем у бананового мороженого, узнайте, какой процент продаж мороженого было произведено каждым отдельным мороженым в прошлом году.

    Банан — 3%, тыква — 7%, а каменистая дорога — 2%

    Пакет с ореховой смесью содержит кешью, фисташки и миндаль. Всего в сумке 1000 орехов, а миндаля на 100 меньше, чем фисташек. Кешью весит 3 г, фисташки — 4 г, миндаль — 5 г. Если сумка весит 3.7 кг, узнайте, сколько орехов каждого вида в мешке.

    Пакет с ореховой смесью содержит кешью, фисташки и миндаль. Изначально в сумке было 900 орехов. Было съедено 30% миндаля, 20% кешью и 10% фисташек, и теперь в сумке осталось 770 орехов. Изначально кешью было на 100 штук больше, чем миндаля. Для начала выясните, сколько орехов каждого типа было в пакете.

    100 миндальных орехов, 200 кешью, 600 фисташек

    Глоссарий

    дополненная матрица
    матрица коэффициентов, примыкающая к столбцу констант, разделенному вертикальной линией в скобках матрицы
    матрица коэффициентов
    матрица, содержащая только коэффициенты из системы уравнений
    Исключение по Гауссу
    с использованием элементарных операций со строками для получения матрицы в виде строки-эшелона
    главная диагональ
    записи из левого верхнего угла по диагонали в правый нижний угол квадратной матрицы
    рядная форма
    после выполнения строковых операций матричная форма, содержащая единицы по главной диагонали и нули в каждом пробеле ниже диагонали
    эквивалент строки
    две матрицы
    А

    и

    B

    эквивалентны строкам, если один может быть получен из другого путем выполнения основных операций со строками

    строковые операции
    добавление одной строки к другой, умножение строки на константу, перестановка строк и т. Д. С целью получения формы «строка-эшелон»

    Эта работа находится под лицензией Creative Commons Attribution 4.0 Международная лицензия.

    Вы также можете бесплатно скачать по адресу http://cnx.org/contents/[email protected]

    Атрибуция:

    Исключение Гаусса — обзор

    Исключение Гаусса и метод Гаусса-Жордана могут использоваться для решения систем сложных линейных уравнений.

    Для комплексной матрицы ее ранг, пространство строк, обратный (если он существует) и определитель могут быть вычислены с использованием тех же методов, что и для вещественных матриц.

    Комплексная матрица n × n W неособая тогда и только тогда | W | ≠ 0, если ранг ( W ) = n .

    Если W , Z являются комплексными матрицами n × n , то | WZ | = | W || Z |, | W T | = | W | и | W * | = | W¯ | = | W | ¯.

    Если A является комплексной матрицей n × n , то λ∈C является собственным значением для A тогда и только тогда, когда существует ненулевой вектор v∈Cn такой, что Av = λ v .Такой ненулевой комплексный вектор v является собственным вектором для A , связанного с λ .

    Если A является комплексной матрицей n × n , собственные значения A являются комплексными корнями характеристического полинома pA (x) = xIn − A A , который делится на комплексные числа на n линейных факторов. То есть сумма алгебраических кратностей собственных значений A равна n .

    Сложная матрица A диагонализуема тогда и только тогда, когда существует невырожденная комплексная матрица P , такая что P −1 AP = D является диагональной матрицей. Метод диагонализации применяется к комплексным матрицам.

    Комплексная n × n матрица A нельзя диагонализовать, если количество основных собственных векторов, полученных методом диагонализации для A , не равно n .

    Пусть A будет комплексной матрицей со всеми действительными элементами. Если A имеет невещественные собственные значения, тогда A нельзя диагонализовать, если рассматривать его как действительную матрицу, но A можно диагонализовать, если рассматривать как комплексную матрицу.

    Метод исключения Гаусса и матричные методы

    Метод исключения Гаусса и матричные методы

    Система линейных уравнений может
    быть помещены в матричную форму.Каждый
    уравнение становится строкой, и каждое
    переменная становится столбцом. An
    добавлен дополнительный столбец для
    справа. Система
    показаны линейные уравнения и результирующая матрица.

    Система линейных уравнений …

     3x + 2y - 4z = 3
    2х + 3у + 3z = 15
    5х - 3у + г = 14 

    становится расширенной матрицей …

    х y z справа
    3 2 -4 3
    2 3 3 15
    5 -3 1 14

    Цель при решении системы уравнений состоит в том, чтобы по возможности преобразовать расширенную матрицу в сокращенную форму строки-эшелона.

    Есть три элементарных операции со строками, которые вы можете использовать для размещения матрицы в
    приведенная строчно-эшелонированная форма.

    Каждое из требований сокращенной матрицы строка-эшелон может быть удовлетворено с использованием элементарной строки
    операции.

    • Если есть строка со всеми нулями, то она находится внизу матрицы.
      Поменяйте местами две строки матрицы, чтобы переместить строку со всеми нулями вниз.
    • Первый ненулевой элемент любой строки — это единица.Этот элемент называется ведущим.
      Умножьте (разделите) строку на ненулевую константу, чтобы превратить первый ненулевой элемент в
      один.
    • Первая строка любой строки находится справа от первой строки предыдущей строки.
      Умножьте строку на ненулевую константу и добавьте ее в другую строку, заменив эту строку. В
      Суть этой элементарной операции со строками состоит в том, чтобы преобразовать числа в нули. Сделав
      числа под ведущими в ноль, это заставляет первый ненулевой элемент любой строки быть
      справа от ведущей предыдущей строки.
    • Все элементы выше и ниже ведущего равны нулю.
      Умножьте строку на ненулевую константу и добавьте ее в другую строку, заменив эту строку. В
      Смысл этой элементарной операции со строками — преобразовать числа в ноль. Разница здесь в
      что вы очищаете (обнуляете) элементы выше ведущего, а не чуть ниже
      ведущий.

    Что такое поворот?

    Цель поворота — сделать элемент выше или ниже ведущего.
    в ноль.

    «Поворотный элемент» или «сводный элемент» — это элемент в левой части матрицы.
    что вы хотите
    элементы сверху и снизу равны нулю.

    Обычно это единица. Если вы найдете книгу, в которой упоминается поворот, они обычно
    сказать вам, что вы должны повернуться на один. Если ограничиться тремя элементарными рядами
    операций, то это верное утверждение.

    Однако, если вы хотите объединить вторую и третью элементарные операции со строками, вы
    придумать другую строковую операцию (не элементарную, но все еще действующую).

    • Вы можете умножить строку на ненулевую константу и добавить ее к ненулевому кратному другому
      row, заменив эту строку.

    И что? Если вам нужно повернуться на одном, то вам иногда придется использовать второй.
    элементарная операция со строкой и разделите строку на ведущий элемент, чтобы превратить ее в единицу.
    Деление приводит к дробям. Хотя дроби — ваши друзья, у вас меньше шансов ошибиться
    если вы их не используете.

    В чем прикол? Если вы не остановитесь на одном, вы, вероятно, столкнетесь с большими числами. Большинство
    люди готовы работать с большими числами, чтобы избежать дробей.

    Процесс поворота

    Pivoting работает, потому что общее кратное (не обязательно наименьшее
    общее кратное) двух чисел всегда можно найти, умножив
    два числа вместе. Возьмем предыдущий пример и
    очистить первый столбец.

    х y z справа
    3 2 -4 3
    2 3 3 15
    5 -3 1 14

    Полезные советы

    • Хотя вам не нужно поворачиваться на одном, это очень желательно.Переход на единицу означает, что вы умножаете на 1 (что легко сделать).
    • Поворачивать по главной диагонали приятно, но не обязательно.
      Некоторым людям нравится начинать с левого верхнего угла и продвигаться вниз к
      Нижний правый.
    • Пока вы выполняете поворот только один раз для каждой строки и столбца, столбцы, которые
      были очищены, останутся очищенными.
    • Поскольку точка поворота — очистить столбец вращения, выбор
      столбец, в котором уже есть нули, экономит время, потому что у вас нет
      чтобы изменить строку, содержащую ноль.

    Выбор оси

    • Выберите столбец с наибольшим количеством нулей.
    • Использовать строку или столбец только один раз
    • Поверните на единицу, если возможно
    • Поворот по главной диагонали
    • Никогда не поворачивайтесь на ноль
    • Никогда не поворачивайте вправо

    Поскольку в первом ряду никого нет, у нас есть два варианта: либо мы
    первую строку делим на три и работаем с дробями, либо делаем поворот на
    три и получите большие числа.Это вариант, который я собираюсь использовать. Я поверну
    на трех в R 1 C 1 . Обведите его как стержневой элемент. В зависимости от вашего браузера вы
    элементы поворота могут быть обведены красным кружком или просто отмечены знаком * перед ним.

    х y z справа
    * 3 2 -4 3
    2 3 3 15
    5 -3 1 14

    Идея состоит в том, чтобы превратить числа в рамке (желтые) в ноль.Использование комбинированного
    рядная операция
    (это не элементарная операция), это может сделать 3R 2
    2R 1 → R 2 и
    3R 3 — 5R 1 → R 3 .

    Единственная строка, которая не изменяется, — это строка, содержащая элемент поворота (
    3). Весь смысл процесса поворота состоит в том, чтобы обнулить значения в рамке.

    Перепишите сводную строку и очистите (сделайте ноль) сводный столбец.

    х y z справа
    * 3 2 -4 3
    0
    0

    Для замены значений в строке 2 каждый новый элемент получается путем умножения
    элемент, заменяемый во второй строке на 3 и вычитающий в 2 раза элемент в первой
    строка из того же столбца, что и заменяемый элемент.

    Чтобы выполнить поворот, приложите один палец к оси поворота (обведено кружком).
    номер) и один палец на заменяемом элементе.
    Умножьте эти два числа вместе. Теперь поместите один палец
    на номере в рамке в той же строке, что и элемент, который вы
    заменяя и другой палец в поворотном ряду и такой же
    столбец как номер, который вы заменяете. Умножьте эти два
    числа вместе. Возьмите изделие за шарнир и
    вычесть произведение без оси.

    х y z справа
    * 3 2 -4 3
    2 3 3 15
    5 -3 1 14

    Чтобы заменить 3 в R 2 C 2 , вы должны взять 3 (3) — 2 (2) = 9-4 = 5.

    Чтобы заменить 3 в R 2 C 3 , вы должны взять 3 (3) — 2 (-4) = 9 +8 = 17.

    Чтобы заменить 15 в R 2 C 4 , вы должны взять 3 (15) — 2 (3) = 45 — 6 = 39.

    Чтобы заменить -3 в R 3 C 2 , вы должны взять 3 (-3) — 5 (2) = -9-10 = -19.

    Чтобы заменить 1 в R 3 C 3 , вы должны взять 3 (1) — 5 (-4) = 3 + 20 = 23

    Чтобы заменить 14 в R 3 C 4 , вы должны взять 3 (14) — 5 (3) = 42-15 = 27.

    Вот как выглядит процесс.

    х y z справа
    поворотный ряд, копия
    3
    поворотный ряд, копия
    2
    поворотный ряд, копия
    -4
    поворотный ряд, копия
    3
    поворотная стойка, прозрачная
    0
    3 (3) — 2 (2)
    5
    3 (3) — 2 (-4)
    17
    3 (15) — 2 (3)
    39
    поворотная стойка, прозрачная
    0
    3 (-3) — 5 (2)
    -19
    3 (1) — 5 (-4)
    23
    3 (14) — 5 (3)
    27

    Или, если убрать комментарии, матрица после первого поворота выглядит так.

    х y z справа
    3 2 -4 3
    0 5 17 39
    0 -19 23 27

    Пришло время
    повторить весь процесс.Мы проходим и выбираем другое место для поворота. Мы
    хотел бы, чтобы он был на главной диагонали, с единицей или с нулями в столбце.
    К сожалению, у нас не может быть ничего из этого. Но так как мы должны все умножить
    другие числа у оси, мы хотим, чтобы она была маленькой, поэтому мы перейдем к
    5 дюймов R 2 C 2 и очистите 2 и -19.

    х y z справа
    3 2 -4 3
    0 * 5 17 39
    0 -19 23 27

    Начните с копирования вниз сводной строки (2-я строка) и очистки сводного столбца (2-я строка).
    столбец).Ранее очищенные столбцы останутся очищенными.

    х y z справа
    0
    0 * 5 17 39
    0 0

    Вот вычисления, чтобы найти следующее взаимодействие.Обратите особое внимание
    в 3-ю строку, где мы вычитаем значение -19 раз. Поскольку мы вычитаем
    отрицательный, я записал его как плюс 19.

    х y z справа
    5 (3) — 2 (0)
    15
    поворотная стойка, прозрачная
    0
    5 (-4) — 2 (17)
    -54
    5 (3) — 2 (39)
    -63
    поворотный ряд, копия
    0
    поворотный ряд, копия
    5
    поворотный ряд, копия
    17
    поворотный ряд, копия
    39
    ранее погашено
    0
    поворотная стойка, прозрачная
    0
    5 (23) + 19 (17)
    438
    5 (27) + 19 (39)
    876

    И получившаяся матрица.

    х y z справа
    15 0 -54-63
    0 5 17 39
    0 0 438 876 ​​

    Обратите внимание, что все элементы в первой строке кратны 3 и все
    элементы в последней строке кратны 438.Разделим, чтобы сократить ряды.

    х y z справа
    5 0 -18-21
    0 5 17 39
    0 0 1 2

    Это имело дополнительное преимущество, давая нам 1, именно там, где мы хотим, чтобы это было
    вращаться.Итак, мы переместимся на 1 в R 3 C 3 и уберем -18 и 17. Обведите свою точку поворота и поместите остальные числа в рамку.
    этот столбец очистить.

    х y z справа
    5 0 -18-21
    0 5 17 39
    0 0 * 1 2

    Скопируйте сводную строку и очистите сводный столбец.Ранее очищенные столбцы
    останется очищенным до тех пор, пока вы не повернете строку или столбец дважды.

    х y z справа
    0 0
    0 0
    0 0 * 1 2

    Обратите внимание, что каждый раз приходится выполнять меньше вычислений.Вот
    расчеты для этой оси. Опять же, поскольку значение в сводном столбце в
    первая строка -18 и мы вычитаем, я записал это как + 18.

    х y z справа
    1 (5) +18 (0)
    5
    ранее погашено
    0
    поворотная стойка, прозрачная
    0
    1 (-21) + 18 (2)
    15
    ранее погашено
    0
    1 (5) — 17 (0)
    5
    поворотная стойка, прозрачная
    0
    1 (39) — 17 (2)
    5
    поворотный ряд, копия
    0
    поворотный ряд, копия
    0
    поворотный ряд, копия
    1
    поворотный ряд, копия
    2

    И получившаяся матрица.

    х y z справа
    5 0 0 15
    0 5 0 5
    0 0 1 2

    Обратите внимание, что первая и вторая строки кратны 5, поэтому мы можем уменьшить их
    ряды.

    х y z справа
    1 0 0 3
    0 1 0 1
    0 0 1 2

    И окончательный ответ: x = 3, y = 1 и z = 2.Вы также можете написать это как
    упорядоченная тройка {(3,1,2)}.

    Надеюсь, вы заметили, что когда я работал с этим примером, я не следовал подсказкам
    Я дал. Это потому, что я хотел, чтобы вы увидели, что произойдет, если вы не повернетесь
    на один. В исходной матрице был один на главной диагонали, и
    Лучше было бы начать с этого.

    Сводка

    • Подбирайте поворотный элемент с умом.
    • Выбор столбца с нулями означает меньший поворот.
    • Выбор единицы в качестве точки поворота уменьшает числа, упрощает умножение и оставляет
      ненулевые элементы в очищенном столбце такие же (без поворота)
    • Поворот по главной диагонали означает, что вам не придется переключать строки, чтобы поместить матрицу в
      приведенная строчно-эшелонированная форма.
    • Не поворачивайтесь на ноль.
    • Не поворачивайте вправо.
    • Используйте строку или столбец только один раз
    • Возьмите продукт с шарниром за вычетом продукта без шарнира

    Особые случаи

    Если вы получите строку из всех нулей, кроме правой части, значит, у системы нет решения.

    Если вы получаете строку со всеми нулями, а количество ненулевых строк меньше, чем количество
    переменных, то система зависима, у вас будет много ответов, и вам нужно написать свой
    ответ в параметрической форме.


    Содержание: Примечания по алгебре колледжа


    Сайт Расс Фрит

    Линейная алгебра

    — зачем использовать метод исключения Гаусса-Джордана вместо исключения Гаусса, Различия

    Следующий пример, часть
    Поиск последовательности элементарных матриц дополняет идею @ Xoque55


    Целевая матрица
    $$
    \оставил[
    \ begin {array} {cc | cc}
    2 и 4 \\
    1 и 1 \\
    \ end {массив}
    \верно]
    $$

    Используйте элементарные операции со строками для исключения Гаусса.$ \ color {blue} {Blue} $ окрашивание обозначает измененные элементы в выходной матрице.


    Рядная форма эшелона

    Формируют расширенную матрицу

    $
    \оставил[
    \ begin {array} {c | c}
    \ mathbf {A} & b
    \ end {массив}
    \верно]
    знак равно
    \оставил[
    \ begin {array} {cc | c}
    2 и 4 и b_ {1} \\
    1 & 1 & b_ {2} \\
    \ end {массив}
    \верно]
    $

    Нормализовать строку 1:
    $$
    \оставил[
    \ begin {array} {cc}
    \ frac {1} {2} & 0 \\
    0 и 1 \\
    \ end {массив}
    \верно]
    %
    \оставил[
    \ begin {array} {cc | c}
    2 и 4 и b_ {1} \\
    1 & 1 & b_ {2} \\
    \ end {массив}
    \верно]
    знак равно
    \оставил[
    \ begin {array} {cc | c}
    \ color {blue} {1} & \ color {blue} {2} & \ frac {1} {2} b_ {1} \\
    1 & 1 & b_ {2} \\
    \ end {массив}
    \верно]
    $

    Очистить столбец 1
    $$
    \оставил[
    \ begin {array} {rc}
    1 & 0 \\
    -1 и 1 \\
    \ end {массив}
    \верно]
    %
    \оставил[
    \ begin {array} {cc | c}
    \ color {blue} {1} & \ color {blue} {2} & \ frac {1} {2} b_ {1} \\
    1 & 1 & b_ {2} \\
    \ end {массив}
    \верно]
    знак равно
    \оставил[
    \ begin {array} {cr | c}
    1 и 2 & \ frac {1} {2} b_ {1} \\
    \ color {blue} {0} & \ color {blue} {- 1} & b_ {2} — \ frac {1} {2} b_ {1} \\
    \ end {массив}
    \верно]
    $

    Система может быть решена обратной заменой.

    Пониженная форма ступени

    Процесс восстановления
    $$
    %
    \оставил[
    \ begin {array} {c | c}
    \ mathbf {A} & \ mathbf {I}
    \ end {массив}
    \верно]
    %
    \ qquad \ Rightarrow \ qquad
    %
    \оставил[
    \ begin {array} {c | c}
    \ mathbf {E_ {A}} и \ mathbf {R}
    \ end {массив}
    \верно]
    $

    Формируют расширенную матрицу

    $
    \оставил[
    \ begin {array} {c | c}
    \ mathbf {A} & \ mathbf {I}
    \ end {массив}
    \верно]
    знак равно
    \оставил[
    \ begin {array} {cc | cc}
    2 и 4 и 1 и 0 \\
    1 и 1 и 0 и 1 \\
    \ end {массив}
    \верно]
    $

    Нормализовать строку 1:
    $$
    \оставил[
    \ begin {array} {cc}
    \ frac {1} {2} & 0 \\
    0 и 1 \\
    \ end {массив}
    \верно]
    %
    \оставил[
    \ begin {array} {cc | cc}
    2 и 4 и 1 и 0 \\
    1 и 1 и 0 и 1 \\
    \ end {массив}
    \верно]
    знак равно
    \оставил[
    \ begin {array} {cc | cc}
    \ color {blue} {1} & \ color {blue} {2} & \ frac {1} {2} & 0 \\
    1 и 1 и 0 и 1 \\
    \ end {массив}
    \верно]
    $

    Очистить столбец 1
    $$
    \оставил[
    \ begin {array} {rc}
    1 & 0 \\
    -1 и 1 \\
    \ end {массив}
    \верно]
    %
    \оставил[
    \ begin {array} {cc | cc}
    1 и 2 & \ frac {1} {2} & 0 \\
    1 и 1 и 0 и 1 \\
    \ end {массив}
    \верно]
    знак равно
    \оставил[
    \ begin {array} {cr | rc}
    1 и 2 & \ frac {1} {2} & 0 \\
    \ color {blue} {0} & \ color {blue} {- 1} & — \ frac {1} {2} & 1 \\
    \ end {массив}
    \верно]
    $

    Нормализовать строку 2
    $$
    \оставил[
    \ begin {array} {cr}
    1 & 0 \\
    0 & -1 \\
    \ end {массив}
    \верно]
    %
    \оставил[
    \ begin {array} {cc | cr}
    1 и 2 & \ frac {1} {2} & 0 \\
    0 & 1 & \ frac {1} {2} & -1 \\
    \ end {массив}
    \верно]
    знак равно
    \оставил[
    \ begin {array} {cc | cr}
    1 и 2 & \ frac {1} {2} & 0 \\
    \ color {blue} {0} & \ color {blue} {1} & \ frac {1} {2} & -1 \\
    \ end {массив}
    \верно]
    $

    Очистить столбец 2
    $$
    \оставил[
    \ begin {array} {cr}
    1 & -2 \\
    0 и 1 \\
    \ end {массив}
    \верно]
    %
    \оставил[
    \ begin {array} {cc | cr}
    1 и 2 & \ frac {1} {2} & 0 \\
    0 & 1 & \ frac {1} {2} & -1 \\
    \ end {массив}
    \верно]
    знак равно
    \оставил[
    \ begin {array} {cc | rr}
    \ color {blue} {1} & \ color {blue} {0} & — \ frac {1} {2} & 2 \\
    0 & 1 & \ frac {1} {2} & -1 \\
    \ end {массив}
    \верно]
    $$
    Результат
    $$
    \оставил[
    \ begin {array} {c | c}
    \ mathbf {E_ {A}} и \ mathbf {R}
    \ end {массив}
    \верно]
    знак равно
    \оставил[
    \ begin {array} {cc | rr}
    1 & 0 & — \ frac {1} {2} & 2 \\
    0 & 1 & \ frac {1} {2} & -1 \\
    \ end {массив}
    \верно]
    $

    Решение
    $$
    \ mathbf {A} x = b
    \ quad \ Rightarrow \ quad
    х = \ mathbf {A} ^ {- 1} б
    \ quad \ Rightarrow \ quad
    x = \ frac {1} {2} \ left [
    \ begin {array} {rr}
    -1 и 4 \\
    1 & -2 \\
    \ end {массив}
    \верно]
    %
    \оставил[
    \ begin {array} {c}
    б_ {1} \\
    Би 2} \\
    \ end {массив}
    \верно]
    \ quad \ Rightarrow \ quad
    х =
    \оставил[
    \ begin {array} {l}
    — \ frac {1} {2} b_ {1} + 2b_ {2} \\
    \ phantom {-} \ frac {1} {2} b_ {1} — b_ {2} \\
    \ end {массив}
    \верно]
    $

    Произведение матриц приведения

    Произведение последовательности матриц редукции является обратным:
    $$
    % четыре
    \оставил[
    \ begin {array} {cr}
    1 & -2 \\
    0 и 1 \\
    \ end {массив}
    \верно]
    % в третьих
    \оставил[
    \ begin {array} {cr}
    1 & 0 \\
    0 & -1 \\
    \ end {массив}
    \верно]
    % второй
    \оставил[
    \ begin {array} {rc}
    1 & 0 \\
    -1 и 1 \\
    \ end {массив}
    \верно]
    % первый
    \оставил[
    \ begin {array} {cc}
    \ frac {1} {2} & 0 \\
    0 и 1 \\
    \ end {массив}
    \верно]
    знак равно
    \оставил[
    \ begin {array} {rr}
    — \ frac {1} {2} & 2 \\
    \ frac {1} {2} & -1 \\
    \ end {массив}
    \верно]
    знак равно
    \ mathbf {A} ^ {- 1}
    $

    Гаусс Джордан Устранение — Объяснение и примеры

    Метод исключения Гаусса-Джордана — это алгоритм для решения линейной системы уравнений.Мы также можем использовать его, чтобы найти обратную матрицу. Давайте сначала посмотрим на определение:

    Исключение Гаусса Джордана или Гаусса исключение — это алгоритм для решения системы линейных уравнений путем представления ее в виде расширенной матрицы, сокращения ее с помощью операций со строками и выражения системы в сокращенной строке. -эшелонированная форма для нахождения значений переменных.

    В этом уроке мы увидим детали метода исключения Гаусса и того, как решить систему линейных уравнений с помощью метода исключения Гаусса-Жордана.Примеры и практические вопросы будут приведены ниже.

    Что такое метод исключения Гаусса?

    Метод исключения Гаусса — это структурированный метод решения системы линейных уравнений. Таким образом, это алгоритм, и его можно легко запрограммировать для решения системы линейных уравнений. Основная цель исключения Гаусса-Джордана:

    • представить систему линейных уравнений в форме расширенной матрицы
    • затем выполнить операции строки $ 3 $ до тех пор, пока не будет получена сокращенная форма эшелона строк (RREF) достигнуто
    • Наконец, мы можем легко распознать решения из RREF

    Давайте посмотрим, что такое расширенная матричная форма, операции строки $ 3 $, которые мы можем сделать с матрицей, и уменьшенная форма эшелона строк матрицы.

    Расширенная матрица

    Система линейных уравнений показана ниже:

    $ \ begin {align *} 2x + 3y & = \, 7 \\ x — y & = 4 \ end {align *} $

    Мы запишет расширенную матрицу этой системы, используя коэффициенты уравнений и запишет ее в стиле , показанном ниже:

    $ \ left [\ begin {array} {rr | r} 2 & 3 & 7 \\ 1 & -1 & 4 \ end {array} \ right] $

    Пример использования одновременных уравнений $ 3 $ показан ниже:

    $ \ begin {align *} 2x + y + z & = \, 10 \\ x + 2y + 3z & = 1 \\ — x — y — z & = 2 \ end {align *} $

    Представление этой системы в виде расширенной матрицы:

    $ \ left [\ begin {array} {rrr | r} 2 & 1 & 1 & 10 \\ 1 & 2 & 3 & 1 \\ — 1 & — 1 & — 1 & 2 \ end {array} \ right] $

    Операции со строками в матрице

    Есть $ 3 $ Элементарные операции со строками , которые мы можем делать с матрицами.Это не изменит решения системы. Это:

    1. Обмен строками $ 2 $
    2. Умножить строку на ненулевой ($ \ neq 0 $) скаляр
    3. Добавить или вычесть скалярное кратное одной строки из другой строки.

    Форма сокращенного эшелона строк

    Основная цель исключения Гаусса Джордана — использовать операции элементарной строки стоимостью 3 доллара в расширенной матрице, чтобы привести ее к форме сокращенного эшелона строк (RREF). Считается, что матрица находится в сокращенной форме эшелона строк , также известной как каноническая форма строки , если выполняются следующие условия $ 4 $:

    1. Строки с нулевыми записями (все элементы этой строки равны 0 $. s) находятся внизу матрицы.
    2. Ведущая запись (первая ненулевая запись в строке) каждой ненулевой строки соответствует справа ведущей записи строки непосредственно над ней.
    3. Начальная запись в любой ненулевой строке — $ 1 $.
    4. Все записи в столбце, содержащем начальную запись ($ 1 $), нулевые.

    Как выполнить исключение Гаусса-Джордана

    В методе исключения Гаусса-Джордана нет каких-либо определенных шагов, но алгоритм ниже описывает шаги, которые мы выполняем, чтобы прийти к сокращенной форме эшелона строк расширенной матрицы.

    1. Поменяйте местами строки так, чтобы все строки с нулевыми записями находились внизу матрицы.
    2. Поменяйте местами строки так, чтобы строка с самой большой левой цифрой находилась наверху матрицы.
    3. Умножьте верхнюю строку на скаляр, который преобразует ведущую запись верхней строки в $ 1 $ (если ведущей записью верхней строки является $ a $, умножьте ее на $ \ frac {1} {a} $, чтобы получить $ 1 $).
    4. Добавить или вычесть кратные числа верхней строки из других строк, чтобы все записи в столбце ведущей записи верхней строки были нулями.
    5. Выполните шаги $ 2 — 4 $ для следующей крайней левой ненулевой записи , пока все ведущие записи каждой строки не будут равны 1 $.
    6. Поменяйте местами строки так, чтобы ведущая запись каждой ненулевой строки находилась справа от ведущей записи строки непосредственно над ней

    На первый взгляд, запомнить / запомнить шаги не так просто. Это вопрос решения нескольких проблем, пока вы не освоитесь с процессом. Существует также фактор интуиции , который играет B-I-G роль в выполнении исключения Гаусса Джордана.

    Давайте рассмотрим несколько примеров, чтобы пояснить процесс решения системы линейных уравнений с помощью метода исключения Гаусса-Джордана .

    Пример 1

    Решите систему, показанную ниже, используя метод исключения Гаусса Джордана:

    $ \ begin {align *} {- x} + 2y & = \, {- 6} \\ { 3x} — 4y & = {14} \ end {align *} $

    Решение

    Первый шаг — написать расширенную матрицу системы.Мы показываем это ниже:

    $ \ left [\ begin {array} {r r | r} — 1 & 2 & — 6 \\ 3 & -4 & 14 \ end {array} \ right] $

    Теперь наша задача состоит в том, чтобы преобразовать матрицу в сокращенную форму эшелона строк (RREF), выполнив команду $ 3 $ элементарные операции со строками.

    У нас есть расширенная матрица:

    $ \ left [\ begin {array} {r r | r} — 1 & 2 & — 6 \\ 3 & — 4 & 14 \ end {array} \ right] $

    Шаг 1:

    Мы можем умножить первую строку на $ — 1 $, чтобы получить ведущий вход $ 1 $.Показано ниже:

    $ \ left [\ begin {array} {r r | r} 1 & — 2 & 6 \\ 3 & — 4 & 14 \ end {array} \ right] $

    Шаг 2:

    Теперь мы можем умножить первую строку на $ 3 $ и вычесть ее из второй ряд. Показано ниже:

    $ \ left [\ begin {array} {r r | r} 1 & -2 & 6 \\ {3 — (1 \ times 3)} & {-4 — (-2 \ times 3)} & {14 — (6 \ times 3)} \ end {array} \ справа] $

    $ = \ left [\ begin {array} {rr | r} 1 & — 2 & 6 \\ 0 & 2 & — 4 \ end {array} \ right] $

    У нас есть $ 0 $ как первая запись второй строки.

    Шаг 3:

    Чтобы сделать вторую запись второй строки $ 1 $, мы можем умножить вторую строку на $ \ frac {1} {2} $. Показано ниже:

    $ \ left [\ begin {array} {r r | r} 1 & — 2 & 6 \\ {\ frac {1} {2} \ times 0} & {\ frac {1} {2} \ times 2} & {\ frac {1} {2} \ times — 4} \ end {array} \ right] $

    $ = \ left [\ begin {array} {rr | r} 1 & — 2 & 6 \\ 0 & 1 & — 2 \ end {array} \ right] $

    Шаг 4:

    Мы почти у цели!

    Вторая запись первой строки должна быть $ 0 $.Для этого мы умножаем вторую строку на $ 2 $ и добавляем ее к первой строке. Показано ниже:

    $ \ left [\ begin {array} {r r | r} {1 + (0 \ times 2)} & {- 2 + (1 \ times 2)} & {6 + (- 2 \ times 2)} \\ 0 & 1 & — 2 \ end {array} \ справа] $

    $ = \ left [\ begin {array} {rr | r} 1 & 0 & 2 \\ 0 & 1 & — 2 \ end {array} \ right] $

    Это сокращенный ряд строк , , , форма . Из расширенной матрицы мы можем написать два уравнения (решения):

    $ \ begin {align *} x + 0y & = \, 2 \\ 0x + y & = -2 \ end {align *} $

    $ \ begin {align *} x & = \, 2 \\ y & = — 2 \ end {align *} $

    Таким образом, решение системы уравнений: $ x = 2 $ и $ y = — 2 $.

    Пример 2

    Решите систему, показанную ниже, используя метод исключения Гаусса Джордана:

    $ \ begin {align *} x + 2y & = \, 4 \\ x — 2y & = 6 \ end { align *} $

    Решение

    Запишем расширенную матрицу системы уравнений:

    $ \ left [\ begin {array} {rr | r} 1 & 2 & 4 \\ 1 & — 2 & 6 \ end {array} \ right] $

    Теперь мы выполняем элементарные операции со строками над этой матрицей, пока не получим сокращенную форму эшелона строк.

    Шаг 1:

    Умножаем первую строку на $ 1 $, а затем вычитаем ее из второй строки. По сути это вычитание первой строки из второй:

    $ \ left [\ begin {array} {r r | r} 1 & 2 & 4 \\ 1 — 1 & — 2 — 2 & 6 — 4 \ end {array} \ right] $

    $ = \ left [\ begin {array} {r r | r} 1 & 2 & 4 \\ 0 & — 4 & 2 \ end {array} \ right] $

    Шаг 2:

    Мы умножаем вторую строку на $ — \ frac {1} {4} $, чтобы получить вторая запись строки, $ 1 $:

    $ \ left [\ begin {array} {rr | r} 1 и 2 и 4 \\ 0 \ times — \ frac {1} {4} & — 4 \ times — \ frac {1} {4} & 2 \ times — \ frac {1} {4} \ end {массив} \ right] $

    $ = \ left [\ begin {array} {rr | r} 1 & 2 & 4 \\ 0 & 1 & — \ frac {1} {2} \ end {array} \ right] $

    Шаг 3:

    Наконец, мы умножаем вторую строку на $ — 2 $ и добавьте его в первую строку, чтобы получить уменьшенную форму эшелона строк этой матрицы:

    $ \ left [\ begin {array} {rr | r} 1 + (- 2 \ times 0) & 2+ (- 2 \ times 1) & 4 + (- 2 \ times — \ frac {1} {2}) \\ 0 & 1 & — \ frac {1 } {2} \ end {array} \ right] $

    $ = \ left [\ begin {array} {rr | r} 1 & 0 & 5 \\ 0 & 1 & — \ frac {1} {2} \ end {array} \ right] $

    Это сокращенный ряд строк формы .Из расширенной матрицы мы можем написать два уравнения (решения):

    $ \ begin {align *} x + 0y & = \, 5 \\ 0x + y & = — \ frac {1} {2} \ end {align *} $

    $ \ begin {align *} x & = \, 5 \\ y & = — \ frac {1} {2} \ end {align *} $

    Таким образом, решение системы уравнений составляет $ x = 5 $ и $ y = — \ frac {1} {2} $.

    Практические вопросы

    1. Решите систему, показанную ниже, используя метод исключения Гаусса Джордана:

      $ \ begin {align *} 2x + y & = \, — 3 \\ — x — y & = 2 \ end {align *} $

    2. Решите систему, показанную ниже, используя метод исключения Гаусса Джордана:

      $ \ begin {align *} x + 5y & = \, 15 \\ — x + 5y & = 25 \ end {align *} $

    Ответы

    1. Начнем с написания расширенной матрицы системы уравнений:

      $ \ left [\ begin {array} {rr | r} 2 & 1 & — 3 \\ — 1 & — 1 & 2 \ end {array} \ right] $

      Теперь мы выполняем элементарные операции со строками, чтобы прийти к нашему решению.

      Первый,
      Мы меняем знаки второй строки местами. Итак, имеем:
      $ \ left [\ begin {array} {r r | r} 1 & 1 & — 2 \\ 2 & 1 & — 3 \ end {array} \ right] $
      Во-вторых,
      Мы дважды вычитаем первую строку из второй строки:
      $ \ left [\ begin {array} { rr | r} 1 & 1 & — 2 \\ 2 — (2 \ times 1) & 1 — (2 \ times 1) & — 3 — (2 \ times — 2) \ end {array} \ right] $
      $ = \ left [\ begin {array} {rr | r} 1 & 1 & — 2 \\ 0 & — 1 & 1 \ end {array} \ right] $
      В-третьих,
      Мы инвертируем вторую строку, чтобы получить:
      $ = \ left [\ begin {array} {rr | r} 1 & 1 & — 2 \\ 0 & 1 & — 1 \ end {array} \ right] $
      Наконец,
      Мы вычитаем вторую строку из первой и получаем:
      $ = \ left [\ begin { массив} {rr | r} 1 & 0 & — 1 \\ 0 & 1 & — 1 \ end {array} \ right] $

      Из этой расширенной матрицы мы можем написать два уравнения (решения):

      $ \ begin {align *} x + 0y & = \, — 1 \\ 0x + y & = — 1 \ end {align *} $

      $ \ begin {align *} x & = \, — 1 \\ y & = — 1 \ end {align *} $

      Таким образом, решение системы уравнений: $ x = — 1 $ и $ y = — 1 $.

    2. Расширенная матрица системы:
      $ \ left [\ begin {array} {rr | r} 1 & 5 & 15 \\ — 1 & 5 & 25 \ end {array} \ right] $
      Давайте приведите эту матрицу к приведенной форме эшелона строк и найдите решение системы.

      Сначала
      Отмените первую строку, затем вычтите ее из второй строки, чтобы получить:
      $ \ left [\ begin {array} {rr | r} 1 & 5 & 15 \\ — 1 — (- 1) & 5 — (- 5) & 25 — (- 15) \ end {array} \ right] $
      $ = \ left [\ begin {array} {rr | r} 1 & 5 & 15 \\ 0 & 10 & 40 \ end {array} \ right] $
      Second,
      Разделите вторую строку на $ 10 $, чтобы получить:
      $ \ left [\ begin {array} {rr | r} 1 & 5 & 15 \\ 0 & 1 & 4 \ end {array} \ right] $
      Затем
      Умножьте вторую строку на $ 5 $ и вычтите ее из первой строки, чтобы получить окончательное решение:
      $ \ left [\ begin {array} {rr | r} 1 — (5 \ times 0) & 5 — (5 \ times 1) & 15 — (5 \ times 4) \\ 0 & 1 & 4 \ end {array} \ right] $
      $ = \ left [ \ begin {array} {rr | r} 1 & 0 & — 5 \\ 0 & 1 & 4 \ end {array} \ right] $
      Это сокращенная форма эшелона строк (RREF).Из этой расширенной матрицы мы можем написать два уравнения (решения):

      $ \ begin {align *} x & = \, — 5 \\ y & = 4 \ end {align *} $

      Таким образом, решение системы уравнений $ x = — 5 $ и $ y = 4 $.

    Предыдущий урок | Главная страница | Следующий урок

    Часть 6: Исключение Гаусса. Исключение Гаусса — это алгоритм… | Авниш | Линейная алгебра

    Метод исключения Гаусса — это алгоритм решения системы линейных уравнений.Он назван в честь немецкого математика Карла Фридриха Гаусса.

    Карл Фридрих Гаусс

    Он аналогичен методу исключения, описанному ранее.

    Для выполнения исключения Гаусса:

    1. Создаем расширенную матрицу коэффициентов и констант данной системы линейных уравнений.
    2. Выбираем нашу pivot (это первый элемент по диагонали). Затем мы пытаемся уменьшить все элементы под ним (до «0»), используя pivot.

    Мы делаем это, выполняя два вида операций:

    a) Умножение сводной строки (строки сводного элемента) на скалярную величину и вычитание или добавление ее строк под ней.

    b) Перестановка строк (например, строка 2 заменяется строкой 3)

    Затем мы выбираем следующую точку поворота (следующий элемент по диагонали) и уменьшаем элементы под ней.

    3. Разбиваем расширенную матрицу обратно на строковое изображение и выполняем умножение с переменной матрицей. Получаем новые редуцированные уравнения.

    Мы решаем эти уравнения, чтобы получить значения неизвестных (переменных).

    Предположим, что нам нужно найти решение (я) следующей системы уравнений:

    4x + y = 9 → (1)

    2x-y = 3 → (2)

    5x-3y = 7 → ( 3)

    (пример «Одно уникальное решение» из Части 5)

    Шаг 1 (Создание расширенной матрицы):

    Для выполнения исключения Гаусса мы берем изображение строки (1), (2) и (3) .Это будет выглядеть следующим образом:

    Затем мы создаем расширенную матрицу для матрицы коэффициентов и постоянной матрицы.

    Единая матрица со значениями коэффициентов и констант, разделенных пунктирной линией

    Шаг 2 (Исключение):

    Шаг 2A:

    Принимая элемент в верхнем левом углу (первый элемент по диагонали) в качестве стержня, мы стремимся исключить ( уменьшить до «0») все элементы под ним. Другими словами, мы должны преобразовать каждый элемент в столбце 1 в «0», кроме pivot.

    Сводной элемент будет выделен красным, а элементы, которые нужно исключить, — синим.

    Итак, мы умножаем первую строку на скаляр 1/2 и вычитаем ее из второй строки.

    Элемент в строке 2 и столбце 1 исключается.

    Затем мы умножаем первую строку на скаляр 5/4 и вычитаем из третьей строки.

    Элемент в строке 3 и столбце 1 исключен.

    Теперь все элементы в первом столбце равны «0», кроме точки поворота.

    Шаг 2B:

    Теперь следующий элемент по диагонали (второй столбец второй строки) установлен как опорный, и мы стремимся удалить все элементы под ним.

    Pivot выделен красным.

    Итак, мы умножаем вторую строку на скаляр 17/6 и вычитаем ее из третьей строки.

    Элемент в строке 3 и столбце 2 исключен.

    Результат — верхняя треугольная матрица.

    Текущее состояние расширенной матрицы называется эшелоном строк формы .

    Шаг 3 (обратная подстановка):

    Теперь мы конвертируем форму эшелона строки обратно в изображение строки.

    У нас было аналогичное уравнение на этапе 1

    При умножении мы получаем:

    Мы составляем уравнения из этих

    4x + y = 9 → (4)

    -3y / 2 = -3/2 → (5)

    Решая (5) относительно «y», получаем:

    y = 1

    Теперь подставляем y = 1 в (4):

    4x + 1 = 9

    4x = 8

    x = 2

    Итак, мы получаем x = 2 и y = 1, именно то, что мы получили, когда решали через изображение строки и изображение столбца в Части 5.

    Теперь применим тот же алгоритм еще в двух случаях (бесконечно много решений и нет решения).

    Бесконечно много решений

    Мы возьмем тот же пример, что и в Части 5. То есть:

    x + 2y = 4 → (6)

    2x + 4y = 8 → (7)

    Шаг 1 (Создание дополненного матрица):

    Строковое изображение (6) и (7) Расширенная матрица строчного изображения выше

    Шаг 2 (Исключение):

    Принятие первого диагонального элемента («1») в качестве опорного.

    Pivot выделен красным, и мы должны удалить все элементы под ним (синим). Чтобы исключить «2», мы дважды вычитаем строку 1 из строки 2 Теперь последняя строка полностью заполнена 0

    Мы больше не делаем поворота так как исключать нечего.

    Шаг 3 (обратная подстановка):

    Мы конвертируем форму эшелона строки обратно в изображение строки:

    После этого мы умножаем ее и получаем новые уравнения

    x + 2y = 4 → (8)

    Уравнение (6) и уравнения (8) такие же, и у нас есть только одно уравнение после исключения, но два неизвестных («x» и «y»).

    Существует множество значений, которыми можно заменить x и y, чтобы удовлетворить (8).

    Нравится, x = 0 и y = 2. Подставляя в уравнение (8), получаем:

    0 + 2 (2) = 4

    4 = 4

    Или x = 1 и y = 1.5. Подставляя в уравнение (8), получаем:

    1 + 2 (1.5) = 4

    1 + 3 = 4

    4 = 4

    Таким образом, система уравнений (6) и (7) имеет бесконечно много решений.

    Нет решения

    Рассмотрим систему линейных уравнений следующим образом:

    x + y = 4 → (9)

    x + y = 8 → (10)

    xy = 0 → (11)

    Применение гауссиана Устранение.

    Шаг 1 (Создание расширенной матрицы):

    Строковое изображение (9), (10) и (11) Расширенная матрица коэффициентов и констант

    Шаг 2 (Исключение):

    Принятие первого диагонального элемента в качестве точки поворота

    Мы выполняем следующие две операции:

    и матрица, которую мы получаем:

    У нас все еще нет формы эшелона строк (верхняя треугольная матрица).

    Итак, мы выполняем обмен строк (который также является вариантом на этапе исключения из метода исключения по Гауссу):

    Замена строки 3 на строку 2 Форма эшелона строки

    Шаг 3 (обратная подстановка):

    Форма эшелона строк преобразована обратно в изображение строки

    Уравнения, которые мы получаем после умножения матриц выше:

    x + y = 4 → (12)

    -2y = -4 → (13)

    Решая уравнение (13) относительно «y», получаем:

    y = 2

    Подставляя y = 2 в уравнение (12), получаем:

    x + 2 = 4

    x = 2

    Чтобы подтвердить, что x = 2 и y = 2 является решением, мы подставляем их в систему уравнений i.е. (9), (10) и (11).

    Подставляя в (9), получаем:

    2 + 2 = 4

    4 = 4

    x = 2 и y = 2, удовлетворяет (9).

    Подставляя в (10), получаем:

    2 + 2 = 8

    4 ≠ 8, это не удовлетворяет (10).

    Следовательно, x = 2 и y = 2 не является решением (9), (10) и (11), и не существует решения этой системы линейных уравнений, как мы видели в прошлой статье.

    Одно решение

    Когда количество неизвестных (переменных) равно количеству уравнения в системе линейных уравнений.

    На примере (1), (2) и (3):

    4x + y = 9 → (1)

    2x-y = 3 → (2)

    5x-3y = 7 → (3)

    Имеется 2 неизвестных («x» и «y») и 3 уравнения ((1), (2) и (3)).

    Двух уравнений было бы достаточно для двух неизвестных.

    Бесконечно много решений

    Когда количество неизвестных превышает количество уравнений.

    В примере (6) и (7):

    x + 2y = 4 → (6)

    2x + 4y = 8 → (7)

    Есть 2 неизвестных («x» и «y») и 2 уравнения ((6) и (7)).

    Добавить комментарий

    Ваш адрес email не будет опубликован. Обязательные поля помечены *